SóProvas



Questões de Psicologia Jurídica: Conceito, Histórico e Atuação


ID
100126
Banca
FCC
Órgão
DPE-SP
Ano
2010
Provas
Disciplina
Psicologia
Assuntos

A liberdade assistida será fixada pelo prazo mínimo de

Alternativas
Comentários
  • O prazo mpinimo da liberdade assistida é de 06 meses, admitindo-se a sua prorrogação.
  • Só para quem quiser saber mais: ler o ECA.
  •  

    Da Liberdade Assistida

    Art. 118. A liberdade assistida será adotada sempre que se afigurar a medida

    mais adequada para o fim de acompanhar, auxiliar e orientar o adolescente.

    § 1º A autoridade designará pessoa capacitada para acompanhar o caso, a qual

    poderá ser recomendada por entidade ou programa de atendimento.

    § 2º A liberdade assistida será fixada pelo prazo mínimo de seis meses, podendo

    a qualquer tempo ser prorrogada, revogada ou substituída por outra medida,

    ouvido o orientador, o Ministério Público e o defensor.

    Fonte: ECA.


ID
100147
Banca
FCC
Órgão
DPE-SP
Ano
2010
Provas
Disciplina
Psicologia
Assuntos

As mudanças sociais e as transformações das normas legais tem trazido diferentes demandas ao psicólogo que trabalha na área jurídica. Outro fator que contribuiu para o aumento da demanda de avaliações psicológicas na esfera jurídica pode ser atribuído

Alternativas
Comentários
  • No livro de Lidia Rosalina sobre o tema (que é seu principal livro) Disputa e Guarda de filhos de casais em divórcio fala que a crescente demanda dos processos jurídicos são, além do DIreito ter a iniciativa de acompanhar as transformações sociais que acontecem nos dias de hoje (e, por isso, demandando de outros profissionais esclarecimentos dessas transformações que influeciam comportamentos e psiquismos) é também como as pessoas procuram o jurídico para mediar seus conflitos.

ID
205069
Banca
FUNDEP (Gestão de Concursos)
Órgão
TJ-MG
Ano
2010
Provas
Disciplina
Psicologia
Assuntos

De acordo com o Estatuto da Criança e do Adolescente (Lei 8.069/90 e alterações posteriores), cabe ao Poder Judiciário, na elaboração de sua proposta orçamentária, prever recursos para manutenção de equipe interprofissional destinada a assessorar a Justiça da Infância e da Juventude.

Sobre a atuação dessa equipe, assinale a afirmativa INCORRETA.

Alternativas
Comentários
  • Incorreta  alternativa A 

    Art. 151. Compete à equipe interprofissional dentre outras atribuições que lhe forem reservadas pela legislação local, fornecer subsídios por escrito, mediante laudos, ou verbalmente, na audiência, e bem assim desenvolver trabalhos de aconselhamento, orientação, encaminhamento, prevenção e outros, tudo sob a imediata subordinação à autoridade judiciária, assegurada a livre manifestação do ponto de vista técnico.

  • LEI Nº 8.069/1990

     

    Art. 151 –  Compete à equipe interprofissional dentre outras atribuições que lhe forem reservadas pela legislação local, fornecer subsídios por escrito, mediante laudos, ou verbalmente, na audiência, e bem assim desenvolver trabalhos de aconselhamento, orientação, encaminhamento, prevenção e outros, tudo sob a imediata subordinação à autoridade judiciária, assegurada a livre manifestação do ponto de vista técnico;

     

    Quem escolheu a busca não pode recusar a travessia - Guimarães Rosa

    ------------------- 

    Gabarito: A


ID
205093
Banca
FUNDEP (Gestão de Concursos)
Órgão
TJ-MG
Ano
2010
Provas
Disciplina
Psicologia
Assuntos

Em relação à atuação do psicólogo nos processos judiciais em Direito de Família, a literatura sobre o tema se divide entre autores e pesquisadores que defendem o exercício da função pericial por meio dos métodos e técnicas de avaliação psicológica e aqueles que se posicionam contrários à atuação do psicólogo como perito stricto sensu nesses processos.

Sobre esse tema, relevante para a atuação do psicólogo no Tribunal de Justiça, analise as seguintes afirmativas.

I. Para os defensores da atuação pericial, a questão é conseguir focalizar o objetivo do trabalho na demanda jurídica que é endereçada ao psicólogo e avaliar as competências individuais e a qualidade do relacionamento entre os membros do grupo familiar, para emitir um laudo que possa contribuir efetivamente para o deslinde da questão processual.

II. Os críticos da atuação estritamente pericial do psicólogo afirmam que a perícia, segundo o Código Civil e o Código de Processo Civil e seus mais importantes intérpretes na literatura brasileira, é procedimento de produção de verdade que, em relação aos conflitos familiares transformados em processos judiciais, tende a definir e reproduzir padrões de comportamento idealizados, normatizando-os. Por isso, esses autores entendem que no trabalho com as questões que emergem dos conflitos familiares, o objetivo deve ser intervir no conflito apresentado e não simplesmente avaliar e relatar, pois dessa forma estaria sendo considerado o sofrimento das pessoas envolvidas e não somente a demanda jurídica.

III. As definições de funções do psicólogo que atua junto à Justiça como servidor, por exemplo, a definição de funções da equipe multidisciplinar nos artigos 150 e 151 do Estatuto da Criança e do Adolescente (Lei 8.069 e suas modificações posteriores) ou mesmo as atribuições do cargo de psicólogo judicial definidas pelo Tribunal de Justiça de Minas Gerais, não restringem esta atuação ao procedimento de perícia tal como estabelecido pelo Código Civil e Código do Processo Civil.

A partir dessa análise, pode-se concluir que estão CORRETAS

Alternativas
Comentários
  • Letra D: todas estão corretas


ID
229249
Banca
FCC
Órgão
TJ-AP
Ano
2009
Provas
Disciplina
Psicologia
Assuntos

A sociedade necessita manter-se à distância dos seus próprios infortúnios, gerando um quadro severo de marginalização e exclusão social. Assim, tem-se que no trabalho nos abrigos de crianças e adolescentes, é de suma importância

Alternativas
Comentários
  • proporcionar um espaço terapêutico aos cuidadores, para que estabeleçam com clareza os limites frente ao envolvimento e o distanciamento na relação com os abrigados.


ID
229252
Banca
FCC
Órgão
TJ-AP
Ano
2009
Provas
Disciplina
Psicologia
Assuntos

A sociedade disciplinar se sustenta num sistema de regras e sanções que é aplicado aos que desviam da norma. O hospital psiquiátrico e a prisão são partes de um mesmo modelo de sociedade que formam uma rede institucional que tem como função controlar o tempo e os corpos dos indivíduos. Pode-se atribuir esse entendimento a

Alternativas
Comentários
  • Foucault é amplamente conhecido pelas suas críticas às instituições sociais, especialmente à psiquiatria, à medicina, às prisões, e por suas ideias e da evolução da história da sexualidade, as suas teorias gerais relativas à energia e à complexa relação entre poder e conhecimento, bem como para estudar a expressão do discurso em relação à história do pensamento ocidental. Fonte: wikipedia
    Além desses temas, a obra de Foucault também aborda amplamente as relações de poder e as sociedades disciplinares.
  • Faz parte do conceito de INSTITUIÇÕES TOTAIS, de Foucalt. (vide também a obra: Manicômios, Prisões e Conventos, do autor).
  • Questão dada


ID
314170
Banca
FCC
Órgão
TRT - 1ª REGIÃO (RJ)
Ano
2011
Provas
Disciplina
Psicologia
Assuntos

Um psicólogo do Tribunal foi nomeado para avaliar um indivíduo e, após concluir um processo psicodiagnóstico, deve redigir o relatório psicológico. Considerando o Manual de Elaboração de Documentos Escritos (Resolução CFP no 007/2003), este documento deve conter a seguinte estrutura mínima:

Alternativas
Comentários
  • Letra b)

    3.2. Estrutura

    O relatório psicológico é uma peça de natureza e valor científicos, devendo conter narrativa detalhada e didática, com clareza, precisão e harmonia, tornando-se acessível e compreensível ao destinatário. Os termos técnicos devem, portanto, estar acompanhados das explicações e/ou conceituação retiradas dos fundamentos teórico-filosóficos que os sustentam.

    O relatório psicológico deve conter, no mínimo, 5 (cinco) itens: identificação, descrição da demanda, procedimento, análise e conclusão.

    http://www.pol.org.br/pol/export/sites/default/pol/legislacao/legislacaoDocumentos/resolucao2003_7.pdf


     





ID
481891
Banca
CESPE / CEBRASPE
Órgão
TRT - 5ª Região (BA)
Ano
2008
Provas
Disciplina
Psicologia
Assuntos

Julgue os itens a seguir, acerca de transtornos psicológicos e do
trabalho do psicólogo.

A autópsia psicológica é uma técnica de avaliação especialmente difícil, porque busca estudar as características de um sujeito já morto e a família não pode ser consultada por questões éticas.

Alternativas
Comentários
  • Questão Errada!

    A autópsia psicológica é um procedimento utilizado em casos onde há dúvidas sobre as circunstâncias de uma morte, fazendo-se necessário uma reconstrução do perfil psicológico da vítima e seu estado mental antes do fato juridicamente questionado.

    Podemos considerar um instrumento útil na medida em que ajuda a esclarecer até que ponto a vítima pode ter provocado as circunstâncias que resultaram em sua morte (suicídio, homicídio, acidente); diminuindo o risco de acusações injustas contra outras pessoas ou a própria pessoa morta. 

    A autópsia psicológica é um processo indireto de exploração da personalidade e vida da pessoa falecida: vai se reconstruindo seus sentimentos, seus afetos, seus relacionamentos, suas dificuldades, seus conhecimentos, seu trabalho, além de histórias clínicas, processo judicial (se houver), escritos, gravações, notas suicidas ou cartas da vítima nos meses anteriores ao evento. No geral, seus antecedentes médicos, psicológicos, sociais e judiciais. A palavra utilizada, “indireto”, deve ser destacada: a análise é feita a partir do relato de outras pessoas (familiares e pessoas do convívio da vítima), o que dá margem a uma eterna dúvida quanto a veracidade das informações, já que a subjetividade da pessoa jamais poderá ser alcançada.

    Fonte: http://sanidadeinsana.blogspot.com.br/2007/11/autpsia-psicolgica.html


ID
635182
Banca
CESPE / CEBRASPE
Órgão
SGA-AC
Ano
2006
Provas
Disciplina
Psicologia
Assuntos

A respeito de violência e de vulnerabilidade, assinale a opção incorreta.

Alternativas
Comentários
  • A violencia nao se define pela carência material e nem por todas as outras razoes dadas pela alternativa a.



ID
635191
Banca
CESPE / CEBRASPE
Órgão
SGA-AC
Ano
2006
Provas
Disciplina
Psicologia
Assuntos

A família de um adolescente usuário de drogas ilícitas foi encaminhada para o atendimento psicológico por ordem judicial. Assinale a opção incorreta acerca do procedimento do psicólogo.

Alternativas
Comentários
  • Significado de Envidar: É o fato de concentrar ações em um objetivo definido.
    Empenhar, esforçar.

  • O juiz não quer relatório de sessões e isso também seria anti ético!


ID
635257
Banca
CESPE / CEBRASPE
Órgão
PC-PA
Ano
2007
Provas
Disciplina
Psicologia
Assuntos

Considerando a atuação do psicólogo em psicologia jurídica, assinale a opção correta.

Alternativas
Comentários
  •       O objeto de estudo da psicologia jurídica, assim como toda a psicologia, são os comportamentos que ocorrem ou que possam vir a ocorrer, porém não é todo e qualquer tipo de comportamento. Ela atua somente nos casos uma inter-relação entre o Direito e a Psicologia, como no caso de adoções, violência doméstica, novas maneiras de atuar em instituições penitenciárias, entre outros.

       

          A psicologia jurídica tem que ver tudo de um ponto de vista jurídico? Não, ao contrário, ela tem que transcender tal visão e observa o problema de um ponto de vista psicológico. Não se pode haver uma estagnação neste tipo de relação. Deve repensar se é possível responder, sob o ponto de vista psicológico, a todas as perguntas que lhe são lançadas. 

           

            Segundo França (2004), a outra forma de relação entre Psicologia Jurídica e Direito é a Complementaridade. A psicologia jurídica, como ciência autônoma, produz conhecimento que se relaciona com o conhecimento produzido pelo Direito, incorrendo numa interseção. Portanto há um diálogo, uma interação, uma interação, bem como haverá diálogo com outros saberes como da Sociologia, Criminologia, entre outros.

     

    https://psicologado.com/atuacao/psicologia-juridica/psicologia-juridica  

     

  • O psicólogo da área jurídica deve transcender as solicitações do mundo jurídico. Deve repensar se é possível responder, sob o ponto de vista psicológico, a todas as perguntas que lhe são lançadas, estabelecendo uma relação de complementaridade entre direito e psicologia.

     

    O psicólogo que atua na área jurídica atende à demanda jurídica, como uma psicologia aplicada, cujo objetivo é contribuir para o melhor exercício do direito, em uma relação de subordinação, na qual o saber psicológico está a serviço do mundo jurídico.

     

    A atuação do psicólogo na área jurídica tem por objeto de estudo uma das manifestações da subjetividade, ou seja, o estudo do comportamento, enquanto que o estudo das conseqüências das ações jurídicas sobre o indivíduo é função da psicologia clínica. 

     

    O psicólogo jurídico pode fazer orientações, contribuir para políticas preventivas e estudar os efeitos do mundo jurídico sobre a subjetividade do indivíduo. A perícia constitui a única atividade que não é praticada pelo psicólogo jurídico. errado

     


ID
697093
Banca
FCC
Órgão
TJ-RJ
Ano
2012
Provas
Disciplina
Psicologia
Assuntos

Em se tratando da avaliação psicológica realizada no contexto forense é possível dizer que as entrevistas

Alternativas
Comentários
  • "devem extrapolar o objetivo da investigação do mundo interno do avaliando, para valorizar, também, aspectos de sua realidade objetiva"

  • Segundo Meloy (1991) apud Rovinsky (2013) a entrevista forense  se diferencia das demais por buscar informações que, posteriormente, devem ser transmitidas a um público leigo. Além de desenvolver habilidades para a redação do laudo, o perito deve se preocupar em tomar notas suficientes durante as entrevistas para garantir a confiabilidade de seus achados. 

  • Características da avaliação psicológica forense.

    Quando se busca responder sobre a determinação do fato:

    A avaliação toma um caráter investigativo;

    O objeto de investigação não se limita ao mundo interno, mas envolve fatos da realidade objetiva;

    No mundo interno o foco da investigação não são os conflitos inconscientes, mas o SEGREDO que a criança precisa guardar da violência real que lhe é impingida.

    Fonte: Palestar ministrada  por Rovinski.

    Acesso: http://docs.tjgo.jus.br/comarcas/foruns/seminario/Seminario_violenciasexual_infanciajuventude.pdf

  • a) cada uma delas tem as suas especificidades;

    b) o psicólogo deve estar atento às situações ligadas à dissimulação e simulação do entrevistado;

    c) tanto os fatos do presente como os da história passada do sujeito devem ser levados em conta;

    d) um dos objetivos das entrevistas é infirmar ou confirmar os achados por meio de outros intrumentos de avaliação psicológica numa perspectiva de intervalidação de resultados;

    Quem escolheu a busca não pode recusar a travessia - Guimarães Rosa

    Gabarito: E


ID
697102
Banca
FCC
Órgão
TJ-RJ
Ano
2012
Provas
Disciplina
Psicologia
Assuntos

Uma conceituação teórica mais recente da relação entre Psicologia e Direito é chamada de Jurisprudência Terapêutica conforme ensina Matthew T. Huss na obra Psicologia Forense (2011). NÃO corresponde ao conceito de Jurisprudência Terapêutica:

Alternativas
Comentários
  • Segundo Huss (2011, p. 32), a "jurisprudência terapêutica (JT) foi definida como 'o uso das ciências sociais para estudar até que ponto uma regra ou prática legal promove o bem-estar psicológico e físico das pessoas que ela afeta' (Slobogin, 1996, p. 767). A jurisprudência terapêutica inclui não só o impacto da lei codificada ou da jurisprudência, mas também o processo legal menos formal que pode focar as ações dos juízes ou advogados. Como a JT se espalhou, ela também tem sido aplicada de modo mais geral para sugerir algum outro modo pelo qual a lei possa ser terapêutica (útil de alguma maneira) ou antiterapêutica (detrimental de alguma maneira). Além disso, a aplicação da JT não infere que uma ação particular deva ter algo a ver com psicoterapia ou mesmo a psicologia clínica em geral. Isso significa que a lei pode ter um impacto fora da rotina da culpa ou inocência de um acusado ou a negligência de um acusado em uma causa civil. A JT sugere que a lei importa além das leis de uma sala de audiências e pode ter um impacto profundo na prática da psicologia forense e em nossas vidas que vai muito além do que nós rotineiramente imaginamos.".

    Dessa forma, NÃO corresponde ao conceito de Jurisprudência Terapêutica: 

    d) a lei nunca tem um impacto fora da rotina da culpa ou inocência de um acusado ou a negligência de um acusado em uma causa civil.


    Fonte: Huss, M. T. (2011). O que é Psicologia Forense? Uma introdução. Em Psicologia Forense (Cap. 1, pp. 21-40). Porto Alegre: Artmed.


ID
698818
Banca
FCC
Órgão
TRE-SP
Ano
2012
Provas
Disciplina
Psicologia
Assuntos

A divergência que é o núcleo de um conflito pode ser factual ou hermenêutica. A divergência hermenêutica dá- se quando

Alternativas
Comentários
  • divergencia factual = fato

    divergência hermenêutica = de interpretação e sentido das palavras 
    Qual alternativa leva a uma divergência hermenêutica? Só pode ser a alternativa a

ID
724288
Banca
FCC
Órgão
TRT - 6ª Região (PE)
Ano
2012
Provas
Disciplina
Psicologia
Assuntos

A privação judicial do direito de um indivíduo para dispor de seus bens ou praticar qualquer ato jurídico é conhecida como

Alternativas
Comentários
  • Imputabilidade é a possibilidade de atribuir a um indivíduo a responsabilidade por uma infração

    curatela é um instituto jurídico pelo qual o curador tem o encargo imposto pelo juiz de cuidar dos interesses de outrem que se encontra incapaz de fazê-lo

    interdição: medida de amparo criada pela legislação civil; por meio do qual a pessoa é declarada civilmente incapaz, total ou

    parcialmente, para a prática dos atos da vida civil,

  • Respsota correta

    Letra C

     


ID
724294
Banca
FCC
Órgão
TRT - 6ª Região (PE)
Ano
2012
Provas
Disciplina
Psicologia
Assuntos

Algumas perspectivas teóricas tentam demonstrar a for-te possibilidade da transmissão intergeracional da violência. Uma dessas teorias é denominada aprendizagem social (Albert Bandura) que, dentre outras coisas, explica que

Alternativas
Comentários
  • Resposta: Letra d

    a) Incorreta. Ao contrário, Bandura sugere que uma das formas de aprendizagem se dá através da observação (aprendizagem vicariante).

    b) Incorreta. A resiliência seria determinada pela forma que os filhos enfrentariam tais situações estressoras, o que varia para cada indivíduo.

    c) Incorreta. É a autoeficácia que determinará os efeitos a tal exposição. Nas palavras de Bandura: "Não são as condições estressantes de vida per si, mas a incapacidade que se sente para lidar com elas que produz os efeitos biológicos nocivos" (1995, p.26).

    d) Correta. Durante a infância, a modelação limita-se à imitação direta, especialmente de seus pais e professores.

    e) Incorreta. As experiências iniciais de construção de eficácia são centradas nos progenitores, através da aprendizagem observacional.


ID
724309
Banca
FCC
Órgão
TRT - 6ª Região (PE)
Ano
2012
Provas
Disciplina
Psicologia
Assuntos

Para a solicitação de participação no Programa Federal de Assistência a Vítimas e a Testemunhas Ameaçadas instituído pela Lei no 9.807 de 13 de Julho de 1999, que estabelece normas para a organização e a manutenção de programas especiais de proteção a vítimas e a testemunhas ameaçadas, o interessado deverá apresentar

Alternativas
Comentários
  • B) CorretaArt. 5º, §2º, II da Lei 9.807/99
    Art. 5º. A solicitação objetivando o ingresso no programa poderá ser encaminhada ao órgão executor: 
    I - pelo interessado 
    (...)
    §2º Para fins de instrução do pedido, o órgão executor poderá solicitar, com a aquiescência do interessado: 
    (...)
    II - exames ou pareceres técnicos sobre a sua personalidade, estado físico ou psicológico.

  • QUESTÃO QUE ERA PASSÍVEL DE ANULAÇÃO

    ====

    O Art. 5º, §2º, II da Lei 9.807/99, prevê:

    "Para fins de instrução do pedido, o órgão executor poderá solicitar, com a aquiescência do interessado:
    (...)
    II - exames ou pareceres técnicos sobre a sua personalidade, estado físico ou psicológico."

    ====

    E o enunciado da questão afirma: "(...) o interessado deverá apresentar ".

    = O examinador não leu a lei.

     

     

  • Para a solicitação de participação no Programa Federal de Assistência a Vítimas e a Testemunhas Ameaçadas instituído pela Lei no 9.807 de 13 de Julho de 1999, que estabelece normas para a organização e a manutenção de programas especiais de proteção a vítimas e a testemunhas ameaçadas, o interessado deverá apresentar exames ou pareceres técnicos sobre a sua personalidade, estado físico ou psicológico.

    lei 9.807/99

    Art. 5º. A solicitação objetivando o ingresso no programa poderá ser encaminhada ao órgão executor: 

    I - pelo interessado 

    (...)

    §2º Para fins de instrução do pedido, o órgão executor poderá solicitar, com a aquiescência do interessado: 

    (...)

    II - exames ou pareceres técnicos sobre a sua personalidade, estado físico ou psicológico.

  • letra B !


ID
724312
Banca
FCC
Órgão
TRT - 6ª Região (PE)
Ano
2012
Provas
Disciplina
Psicologia
Assuntos

Maria Helena Diniz, na obra de sua autoria denominada Código Civil Comentado (2009), pautada no artigo 1.767 do Código Civil, define Curatela como sendo o encargo público cometido, por lei, a alguém para reger e defender uma pessoa e administrar os bens de maiores incapazes, que, por si sós, não estão em condições de fazê-lo, em razão de enfermidade ou deficiência mental. NÃO estão sujeitos à Curatela:

Alternativas
Comentários
  • Alguém?

  • Letra A. Sua causa primária está relacionada com desvio de conduta, aspectos caracterológicos . F.60.2 , não possuem alterações relacioandas à incapacidade de raciocínio e outros aspectos intelectuais. 

  •  d) alcoólatras eventuais.

    Entendi que a pessoa bebe eventualmente, ou por períodos o que não caracteriza perfil para curatela. Os casos de interdição precisam de uma deficiência, lesão ou condição pessoal que se mantenha de forma permanente.

     

    "A paciência é uma segunda coragem"   Kafka

  • Maria Helena Diniz, na obra de sua autoria denominada Código Civil Comentado (2009), pautada no artigo 1.767 do Código Civil, define Curatela como sendo o encargo público cometido, por lei, a alguém para reger e defender uma pessoa e administrar os bens de maiores incapazes, que, por si sós, não estão em condições de fazê-lo, em razão de enfermidade ou deficiência mental. NÃO estão sujeitos à Curatela alcoólatras eventuais


ID
724321
Banca
FCC
Órgão
TRT - 6ª Região (PE)
Ano
2012
Provas
Disciplina
Psicologia
Assuntos

A prática transdisciplinar significa

Alternativas
Comentários
  • uma forma de trabalho isolada com uma coordenação apenas administrativa. errado

     

    a apropriação de contribuições de outras disciplinas de forma subordinada. errado

     

    diversos campos do saber atuando sem cooperação ou troca de informações. errado

     

    a estabilização de um campo teórico, aplicado ou disciplinar, do tipo novo ou mais amplo. verdade

     

    eu considero que essa é correta também. Alguem tem alguma explicação? não ter preocupação com a criação de uma axiomática própria - um sistema axiomático é dito consistente se não há contradição, i.e., não possui capacidade de derivar a afirmação e negação de uma mesma sentença.

     

  • Ju, o erro da alternativa E está em dizer que não é uma preocupação da transdisciplinaridade formar uma única rede de conhecimento, como se não fosse um dos seus objetivos, quando na verdade é o seu principal.

    Espero ter ajudado! :)


ID
724327
Banca
FCC
Órgão
TRT - 6ª Região (PE)
Ano
2012
Provas
Disciplina
Psicologia
Assuntos

A Resolução CFP 8/2010, que dispõe sobre a “Atuação do Psicólogo como Perito e Assistente Técnico no Poder Judiciário”, estabelece em seu capítulo IV, artigo 10, que ao psicólogo que atua como psicoterapeuta das partes é

Alternativas
Comentários
  • CAPÍTULO IV
    O PSICÓLOGO QUE ATUA COMO PSICOTERAPEUTA DAS PARTES

    Art. 10 - Com intuito de preservar o direito à intimidade e equidade de condições, é vedado ao psicólogo que esteja atuando como psicoterapeuta das partes envolvidas em um litígio:

    I - Atuar como perito ou assistente técnico de pessoas atendidas por ele e/ou de terceiros envolvidos na mesma situação litigiosa;

    II - Produzir documentos advindos do processo psicoterápico com a finalidade de fornecer informações à instância judicial acerca das pessoas atendidas, sem o consentimento formal destas últimas, à exceção de Declarações, conforme a Resolução CFP Nº 07/2003.

    Parágrafo único - Quando a pessoa atendida for criança, adolescente ou interdito, o consentimento formal referido no caput deve ser dado por pelo menos um dos responsáveis legais.

  • RESOLUÇÃO CFP Nº 008/2010

    Art. 10 - Com intuito de preservar o direito à intimidade e equidade de condições, é vedado ao psicólogo que esteja atuando como psicoterapeuta das partes envolvidas em um litígio:

    I - Atuar como perito ou assistente técnico de pessoas atendidas por ele e/ou de terceiros envolvidos na mesma situação litigiosa;

    Gabarito: C


ID
729208
Banca
FCC
Órgão
MPE-PE
Ano
2012
Provas
Disciplina
Psicologia
Assuntos

Proposta de inquirição destinada à oitiva de crianças apontadas como vítimas ou testemunhas de violência ou maus-tratos. A sala reservada para ouvir a criança vítima de violência é conectada por vídeo e com zoom à sala de audiência onde estão o magistrado, o promotor de justiça, advogados, réu e servidores da Justiça, que podem interagir durante o depoimento. Todo o procedimento é gravado na memória de um computador, transcrito e juntado aos autos, além de ser copiado em mídia de armazenamento digital de dados que é inserida na contracapa do processo. O depoimento é tomado por um técnico entrevistador (profissional da área de psicologia ou do serviço de assistência social). Corresponde

Alternativas
Comentários
  • Lembrando que de acordo com resolução atual, o CFP se manifestou contrariamente ao  "depoimento sem dano", por entender que são práticas de interrogatório e investigação, isto é, não cabem ao psicólogo.

    http://www.pol.org.br/noticias/materia.cfm?id=932&materia=1400 

  • Resposta D

    O chamado Depoimento sem Dano (DSD) consiste na oitiva de crianças e adolescentes em situação de violência. O depoimento é tomado por um técnico (psicólogo ou assistente social) em uma sala especial, conectada por equipamento de vídeo e áudio à sala de audiência, em tempo real. O técnico possui um ponto eletrônico, através do qual o juiz direciona as perguntas a serem feitas à criança. Além disso, o depoimento fica gravado, constando como prova no processo.


    fonte http://www.crprj.org.br/comissoes/justica/depoimento-sem-dano.html

  • NOTA TÉCNICA SOBRE OS IMPACTOS DA LEI Nº 13.431/2017 (depoimento especial) NA ATUAÇÃO DAS PSICÓLOGAS E DOS PSICÓLOGOS

    http://site.cfp.org.br/wp-content/uploads/2018/01/NOTA-TECNICA-N%C2%BA-1_2018_GTEC_CG.pdf

     

    "Daqui a um ano, você vai desejar ter começado hoje" Karen Lamb    
     


ID
732361
Banca
FCC
Órgão
TRF - 2ª REGIÃO
Ano
2012
Provas
Disciplina
Psicologia
Assuntos

A articulação da Psicologia com o Direito teve origem na

Alternativas
Comentários
  • Resposta letra E

      Do ponto de vista histórico, a primeira grande articulação entre a ciência psicológica e o Direito teve origem na avaliação do fidedignidade de testemunhos, fato que contribuiu para o desenvolvimento da Psicologia Experimental no século XIX. No estudo do tema, Mira y López (1967) expõe:


              O testemunho de uma pessoa sobre um acontecimento qualquer depende essencialmente de cinco fatores: a) do modo como percebeu esse acontecimento; b) do modo como sua memoria o conservou; c) do modo como é capaz de evoca-lo; d) do modo como quer expressa-lo; e) do modo como pode expressa-lo.

    RETIRADO DO SEGUINTE ENDEREÇO:
    http://www.trabalhosfeitos.com/ensaios/Reflex%C3%B5es-Em-Torno-Psicologia-Juridica/264102.html
  • A título de curiosidade, 
    A Psicologia do Testemunho visa estudar os depoimentos prestados junto de instâncias de controlo social e, do ponto de vista científico, é um segmento da Psicologia Forense (ou Judiciária) Experimental. Neste sentido, o seu objecto consiste na averiguação da verdade, do erro e da mentira no cenário judicial (Sabaté, Bayés e Munné, 1980, citados por Diges e Alonso-Quecuty, 1993).

    (Retirado do artigo
     Psicologia do testemunho: Contribuição para a aproximação …)
  • Questão pegadinha, pois ela enumera as escolas mais conhecidas da psicologia e coloca a psicologia do testemunho lá embaixo pra confundir o candidato. Temos que tomar cuidado ao resolver questões. 

  • A Psicologia Jurídica emergiu da Psicologia do Testemunho cuja prática, em âmbito internacional, ajudou a consolidar a Psicologia enquanto ciência, dada a necessidade de sua contribuição na comprovação da fidedignidade de testemunhos, principalmente com o surgimento e aplicação dos testes psicológicos, em meados do século XX, assim como o desenvolvimento de estudos sobre os funcionamentos dos interrogatórios, dos delitos, dos falsos testemunhos e falsas memórias etc., colaborando para a criação dos primeiros laboratórios de Psicologia.

    https://www.concursospsicologia.com/artigo-psicologia-juridica


ID
732364
Banca
FCC
Órgão
TRF - 2ª REGIÃO
Ano
2012
Provas
Disciplina
Psicologia
Assuntos

A Resolução do Conselho Federal de Psicologia de no 013/2007 reconhece algumas especialidades como campo de atuação do psicólogo. Assim, tendo como base a citada Resolução, a Psicologia Jurídica

Alternativas
Comentários
  • Art. 3o - As especialidades a serem concedidas são as seguintes:

    I. Psicologia Escolar/Educacional;
    II. Psicologia Organizacional e do Trabalho;
    III. Psicologia de Trânsito;
    IV. Psicologia Jurídica;
    V. Psicologia do Esporte;
    VI. Psicologia Clínica;
    VII. Psicologia Hospitalar;
    VIII. Psicopedagogia;
    IX. Psicomotricidade;
    X. Psicologia Social;
    XI. Neuropsicologia.
  • LETRA C.

     

  • RESOLUÇÃO CFP Nº 013/2007

     

    Art. 3º –  As especialidades a serem concedidas são as seguintes:

     

    I. Psicologia Escolar/Educacional;

    II. Psicologia Organizacional e do Trabalho;

    III. Psicologia de Trânsito;

    IV. Psicologia Jurídica;

    V. Psicologia do Esporte;

    VI. Psicologia Clínica;

    VII. Psicologia Hospitalar;

    VIII. Psicopedagogia;

    IX. Psicomotricidade;

    X. Psicologia Social;

    XI. Neuropsicologia.

     

    Quem escolheu a busca não pode recusar a travessia - Guimarães Rosa

    ------------------- 

    Gabarito: C


ID
732376
Banca
FCC
Órgão
TRF - 2ª REGIÃO
Ano
2012
Provas
Disciplina
Psicologia
Assuntos

A lei incide como limite na sociedade, mas sem desconsiderar que limite é um conceito de ordem simbólica, da ordem da Lei do Pai, e que seu manejo ou sua aplicação vão depender sempre da capacidade de

Alternativas
Comentários
  • A lei incide como limite na sociedade, mas sem desconsiderar que limite é um conceito de ordem simbólica, da ordem da Lei do Pai, e que seu manejo ou sua aplicação vão depender sempre da capacidade de simbolização de seu operador, legislador ou juiz.


ID
732379
Banca
FCC
Órgão
TRF - 2ª REGIÃO
Ano
2012
Provas
Disciplina
Psicologia
Assuntos

O sujeito do Direito é aquele que age consciente de seus direitos e deveres e que segue leis estabelecidas em um dado ordenamento jurídico. Já, para a Psicanálise, o sujeito está assujeitado às leis

Alternativas
Comentários
  • e) regidas pelo inconsciente.
  • O ego e o superego são instâncias do inconsciente, mas não as únicas, faltou o ID . Além disso, o ego, grosso modo, é o que emerge do conflito entre o superego e o id e essas instâncias mesmas são regidas por outros mecanismos inconscientes, por exemplo, o Nome do Pai que será determinante no estabelecimento da função superegóica.

  • Para a psicanálise, o sujeito está submetido ao inconsciente

  • Não seria letra c??


ID
740296
Banca
CEPERJ
Órgão
DEGASE
Ano
2012
Provas
Disciplina
Psicologia
Assuntos

Leia os itens seguintes sobre o atendimento técnico dos psicólogos no trabalho com adolescentes:

I- A qualidade do atendimento e o que ocorre no interior das unidades de internação e privação de liberdade é da responsabilidade dos profissionais que lá trabalham.

II- A atuação do psicólogo é limitada à elaboração de pareceres e relatórios sobre os adolescentes.

III- É dever dos psicólogos intervir e buscar auxilio em instâncias superiores de gestão quando o adolescente se encontra em situação de risco de morte ou grave adoecimento.

IV- A atuação do psicólogo é sempre de neutralidade.

Em relação ao atendimento técnico mencionado, a alternativa certa é:

Alternativas
Comentários
  • Resposta Letra B

    Sobre o item IV:


    Coimbra e Nascimento (2001) fundamentam-se em Foucault para mostrar a impossibilidade de as ações da Psicologia serem neutras ou apolíticas, ressaltando que:

    Para Foucault (1988), os saberes, compreendidos como materialidade, práticas e acontecimentos, são dispositivos políticos articulados com as diferentes formações sociais, inscrevendo - se, portanto, em suas condições políticas. Daí afirmar que não há saber neutro: todo saber é político. (p. 246)

    O que essas autoras querem afirmar é que Foucault denuncia a inexistência de uma essência das coisas, afirmando que os sujeitos com quem a Psicologia trabalha e a própria concepção de Psicologia são construídos historicamente e, portanto, imbuídos de um contexto político (Coimbra & Nascimento, 2001).

    A partir dessa reflexão acerca do distanciamento que historicamente foi se construindo entre Psicologia e política na tentativa de construção de um campo de saber científico, objetiva-se, neste artigo, visualizar os impasses que essa dicotomização provoca na formação de profissionais de Psicologia para trabalhar em políticas públicas, especialmente no que se refere ao Sistema Único de Saúde. Pretende-se evidenciar os contrastes existentes nas demandas ao trabalho de profissionais comprometidos com o cuidado e a promoção de saúde, e não com uma prática orientada a partir da neutralidade e do distanciamento, presentes na forma de se aplicarem os conhecimentos no desenvolvimento das práticas profissionais.

    Fonte: http://www.scielo.br/scielo.php?script=sci_arttext&pid=S1414-98932010000400014



ID
767518
Banca
FCC
Órgão
MPE-AP
Ano
2012
Provas
Disciplina
Psicologia
Assuntos

A avaliação psicológica das famílias no contexto forense deve levar em conta algumas características que a diferem do modelo clínico. Dentre essas diferenças presentes entre um e outro modelo pode-se citar a

Alternativas
Comentários
  • b) relação profissional-cliente; comunicação dos resultados; origem da demanda.


    Relação profissional-cliente: os psicólogos forenses estudam o comportamento humano e procuram aplicar esses princípios para auxiliar o sistema legal (Huss, 2011). Ainda segundo Huss (2011, p. 24), "a prática clínica da psicologia [forense] focaliza, em geral, a avaliação e tratamento dos indivíduos dentro de um contexto legal e inclui conceitos como psicopatia, inimputabilidade, avaliação de risco, danos pessoais e responsabilidade civil".

    Comunicação dos resultados:
     Segundo Meloy (1991) apud Rovinsky (2013) a entrevista forense  se diferencia das demais por buscar informações que, posteriormente, devem ser transmitidas a um público leigo. Além de desenvolver habilidades para a redação do laudo, o perito deve se preocupar em tomar notas suficientes durante as entrevistas para garantir a confiabilidade de seus achados. 

    Origem da demanda: na clínica, geralmente é a pessoa ou familiar que procura a avaliação psicológica e, na forense é, usualmente, o advogado ou o Tribunal.

  • b) relação profissional-cliente; comunicação dos resultados; origem da demanda.

    Relação profissional-cliente: os psicólogos forenses estudam o comportamento humano e procuram aplicar esses princípios para auxiliar o sistema legal (Huss, 2011). Ainda segundo Huss (2011, p. 24), "a prática clínica da psicologia [forense] focaliza, em geral, a avaliação e tratamento dos indivíduos dentro de um contexto legal e inclui conceitos como psicopatia, inimputabilidade, avaliação de risco, danos pessoais e responsabilidade civil".

    Comunicação dos resultados: Segundo Meloy (1991) apud Rovinsky (2013) a entrevista forense se diferencia das demais por buscar informações que, posteriormente, devem ser transmitidas a um público leigo. Além de desenvolver habilidades para a redação do laudo, o perito deve se preocupar em tomar notas suficientes durante as entrevistas para garantir a confiabilidade de seus achados. 

    Origem da demanda: na clínica, geralmente é a pessoa ou familiar que procura a avaliação psicológica e, na forense é, usualmente, o advogado ou o Tribunal.


ID
775102
Banca
FADESP
Órgão
SESPA-PA
Ano
2010
Provas
Disciplina
Psicologia
Assuntos

Cidadania e direitos da cidadania dizem respeito a uma determinada ordem jurídico-política de um país, de um estado, no qual uma Constituição define e garante quem é cidadão, que direitos e deveres ele terá em função de uma série de variáveis tais como

Alternativas

ID
826678
Banca
CESPE / CEBRASPE
Órgão
TJ-RO
Ano
2012
Provas
Disciplina
Psicologia
Assuntos

Com relação à psicologia jurídica, assinale a opção correta.

Alternativas
Comentários
  • Alguém poderia dizer o que há de errado com a alternativa D?
  • As áreas de atuação da psicologia é mais abrangente, tais como violência familiar e infância e juventude não foram citados. Sua competência não se limita apenas oq a questão menciona.
    Parecer, perícia, ações como informar, apoiar, acompanhar, orientar e criar condições que visem a eliminar a opresssão e a marginalização.
  • Gabarito E

    Letra A: Errada. Não contribuiu inicialmente com a área da vitimologia.

    Letra B: Errada. A psicometria não é o campo de maior relevância e mais estudado hoje, foi estudado apenas, contribuindo de alguma forma.

    Letra C: Errada. A psicologia jurídica se insere na área de guarda e tutela muito tardiamente.

    Letra D: Errada. Não é da competência do psicólogo jurídico propor soluções de conflitos (nem o mediador tem esse papel) muito menos prevenir sofrimentos psíquicos graves.

    Letra E: Certa.


    Site: https://psicologado.com/atuacao/psicologia-juridica/atuacao-do-psicologo-no-campo-juridico

  • A competencia dele não é essa Lidiane.

  • Olá Lidiane!

    A resposta D está incorreta porque afirma "propor solução de conflitos". Num litígio, a competência para resolução de um conflito é do juiz. Se um psicólogo age como perito num litígio, não pode propor solução para o conflito; deve o psicólogo perito apenas trazer os esclarecimentos necessários ao juiz sobre processos psicológicos (comportamento, afetos, relações, vínculos, psicopatologia, etc.) que tenham sido objeto do pedido da perícia.

     Quando se trata de uma mediação, compete às partes que estão mediando compor a solução do próprio conflito, nesse caso o psicólogo é somente mediador, o que quer dizer que o papel dele é facilitar um diálogo entre as partes.

  • O termo ''voluntário'' da letra E me deixou um pouco em dúvida.


ID
826681
Banca
CESPE / CEBRASPE
Órgão
TJ-RO
Ano
2012
Provas
Disciplina
Psicologia
Assuntos

A atuação do psicólogo jurídico pode abranger

Alternativas
Comentários

  • Errado  enquanto mediador, uma função interventora, no intuito de solucionar conflitos, focalizando estabelecimento de acordo entre as partes, mesmo que o resgate do canal de comunicação não ocorra. 

    Errado a aplicação de questões psicodiagnósticas e a elaboração de laudos e pareceres relativos às áreas criminal e civil, podendo o psicólogo decidir e opinar sobre o andamento do processo judicial. 

    Errado a criação de redes de assistência a famílias de alto risco, com o foco principal em atendimento conjunto de crianças vítimas de abusos e abusadores, pois o trabalho que envolva toda a família é sempre mais benéfico. 

    Errado o desenlace das dificuldades com as quais o Poder Judiciário, frequentemente, precisa lidar, desde que relacionadas a seu campo de atuação, sem intercâmbio de conhecimento técnico com outros campos. 

    certo a organização do contexto de referência familiar, a fim de que a criança possa se constituir como sujeito e se desenvolver de maneira saudável.

  • O que está errado na letra A não é o fato da mediação ser uma função interventora, o que isso ela é; os erros estão no objetivo de solucionar conflitos ( a mediação facilita o diálogo, promove a reflexão, para que as partes encontrem soluções criativas ou simplesmente não encontrem soluções). A função é facilitar a comunicação, usar o conflito enquanto potencial criativo.

    Na letra C o erro está em "no atendimento conjunto de crianças vítimas de abusos e abusadores"

  • Mariha, o erro da A está sim em intervenção.

    A lei da Mediação - LEI Nº 13.140, DE 26 DE JUNHO DE 2015:

    Art. 2o A mediação será orientada pelos seguintes princípios:

    I - imparcialidade do mediador;


    Na ocasião da prova ainda não tinha a Lei, mas já era um princípio da Mediação a Imparcialidade.
    Bons estudos!
  • gabarito E

    a organização do contexto de referência familiar, a fim de que a criança possa se constituir como sujeito e se desenvolver de maneira saudável.

  • Organização do contexto de referência familiar? Há algum autor de referência que fale sobre isso?


ID
826684
Banca
CESPE / CEBRASPE
Órgão
TJ-RO
Ano
2012
Provas
Disciplina
Psicologia
Assuntos

No que se refere à psicologia jurídica, assinale a opção correta.

Alternativas
Comentários
  • Muito fácil constatar que todas estão erradas, menos a alternativa c.


  • Quanto a questão da expressão da subjetividade, vale lembrar o que Shine aponta, no âmbito jurídico o cliente é o juiz e não as partes do processo.

  • Letra C

    Vale comentar que os estatutos do idoso, por exemplo, não estipula a atuação de psis ou outros profissionais nas varas de idoso (quando houver), e isso faz que os TJs não coloquem psis nessas varas. 

  • O juiz é o cliente, mais o psicólogo é aquele que atua no campo do subjetivo, da subjetividade do sujeito, não pode interferir que ela surja ao longo da avaliação, mas, pode direcionar a mesma para que a questão solicitada pelo juiz seja respondida com vigor 

  • Letra E está errada por que utilizou procedimentos técnicos e éticos. O correto seria: técnicos e científicos

  • Na realidade a letra E está errada porque afirma que "o psicólogo pode utilizar habilidades outras que não aquelas específicas do conhecimento psicológico" sendo que o código de ética é muito claro em relação a isso, vejamos:

    Art 2º Ao psicólogo é vedado:

    • f) Prestar serviços ou vincular o título de psicólogo a serviços de atendimento psicológico cujos procedimentos, técnicas e meios não estejam regulamentados ou reconhecidos pela profissão;

    Gabarito: C


ID
826687
Banca
CESPE / CEBRASPE
Órgão
TJ-RO
Ano
2012
Provas
Disciplina
Psicologia
Assuntos

Com relação à perícia, assinale a opção correta.

Alternativas
Comentários
  • A pericia não é uma atividade essencialmente juridica, existe a pericia em outras aréas como médica, engenharia e outras. 
  • Eu errei esta questão por descuido. A alternativa D (Gabarito) fala de verificação de fatos e incidências inerentes à causa, logo pode-se inferir que o que é desenvolvido é um "estudo" que não decorre da necessidade de avaliação das partes acerca de algum ponto que envolve o litigio, se fosse assim ao juiz seria transmitido o laudo.

  • resposta D) 

    Inerentes a causa, sendo essa a demanda inicial da pericia (uma modalidade de avaliação psi onde há subetendido ou explicito um pedido/questões a serem respondidas, sendo estas feitas pelo Juiz). 

    Foco na causa implica que não se avaliará outras questões não relevantes a causa. Exemplo, em uma questão de licensa médica por depressão, não se avaliara questões sexuais.

  • A perícia (psicológica, médica, contábil...) é orientada para a produção de investigações(averiguação de fatos e incidências) e para comunicação de seus resultados(laudo, pareceres etc.).

  • A lei 4.112 de 27 de agosto de 1962, que dispõe sobre a profissão de psicólogo, afirma que no exercício profissional, entre outras atribuições, cabe ao psicólogo: "Realizar perícias e emitir pareceres sobre a matéria de psicologia" (Art. 4º, n° 6). Por sua vez, o nosso Código de Ética Profissional estabelece, em seus artigos de 18 a 22, os limites que norteiam a relação do psicólogo com a Justiça. Portanto, esta é uma área de atuação legítima do psicólogo. Cabe a ele desenvolver o estudo da personalidade dos litigantes e demais envolvidos nos litígios judiciais. Caso as ilações periciais sejam baseadas em psicodiagnósticos, cabe-lhe também concluir o laudo.

     

  • Achei super difícil, pois, estamos habituados a associar o parecer ao assistente técnico... que não entrega parecer ao juiz mas sim à parte, a qual decide se entregará ao advogado ou não acerca do parecer para compor os autos do processo...

    Mas enfim... nessa questão estão tratando de possibilidades gerais do trabalho do psicólogo nas perícias e não necessariamente de uma lide que envolva necessariamente psicólogo perito e assistente técnico. É isso?


ID
826750
Banca
CESPE / CEBRASPE
Órgão
TJ-RO
Ano
2012
Provas
Disciplina
Psicologia
Assuntos

Estão sujeitos à curatela os

Alternativas
Comentários
  • A diferença básica é que o tutor é responsável pelos incapazes em razão da idade: menores de 18 anos. O curador, por sua vez, é quem representa os interesses de quem é incapaz em virtude de outras causas, que ensejam a incapacidade de uma pessoa após os 18 anos: enfermidade ou deficiência mental que impeça discernimento para a prática desses atos, aqueles que, mesmo por causa transitória, não puderem exprimir sua vontade (internados em coma, por exemplo), ébrios habituais, viciados em tóxicos, os que, por deficiência mental, tenham o discernimento reduzido, e também os excepcionais, sem desenvolvimento mental completo, e também os considerados pródigos (gastadores compulsivos).

    A partir dessa distinção existem uma série de oturas diferenças quanto às regras da tutela e curatela, previstas no Código Civil. Mas são institutos tão próximos que o artigo 1.781 do Código até determina que as regras a respeito do exercício da tutela sa aplicam ao da curatela, salvo quanto às regras específicas sobre o curador.

    Por isso, do menor não há curatela, há tutela. Se, depois da maioridade, existir alguma outra causa que enseje a incapacidade, deverá ser convertida a tutela em curatela, e o tutor em curador, por decisão judicial. Isso porque a incapacidade do menor decorre automaticamente de lei, todo menor precisa de um representante ou assistente. Já a curadoria de um maior depende de decisão judicial que decrete a chamada "interdição" do maior, nomeando-lhe um curador.

    Para detalhes da diferença, confira os artigos 1.728 a 1.783 do Código Civil, está tudo lá.

    http://br.answers.yahoo.com/question/index?qid=20110610103829AA86qp1
  • Acredito que a pergunta ficou ambígua. 
    Creio que eles queriam que marcassem a sujeito absolutamente incapaz, que é representado pelo ser curador. Porém, o alcoólatra (ébrio habitual) é classificado como relativamente incapaz, sendo assistido pelo curador. 


    "Art. 1.767. Estão sujeitos a curatela:

    I - aqueles que, por enfermidade ou deficiência mental, não tiverem o necessário discernimento para os atos da vida civil;

    II - aqueles que, por outra causa duradoura, não puderem exprimir a sua vontade;

    III - os deficientes mentais, os ébrios habituais e os viciados em tóxicos;

    IV - os excepcionais sem completo desenvolvimento mental;

    V - os pródigos."


  • realmente a questão foi mal formulada.

  • Três respostas possíveis de acordo com o artigos: a), b) e c).

     

    "O sucesso é ir de fracasso em fracasso sem perder entusiasmo"


ID
832126
Banca
PUC-PR
Órgão
DPE-PR
Ano
2012
Provas
Disciplina
Psicologia
Assuntos

Ao abordar hipóteses a respeito dos fatores que contribuem para que um indivíduo venha a delinquir podemos considerar:


I. Predisposição genética – O pesquisador Hare sustenta que a prevalência de psicopatas na população carcerária seria de 15 a 20%, enquanto que na população em geral é em torno de 01%. Entretanto Fernandes e Fernandes (1995) assinalam que “ parece irrecusável que existe uma contribuição genética para quase toda forma de comportamento, mas não é absolutamente verdadeiro que o comportamento específico dos seres humanos seja determinado apenas geneticamente.

II. O lar: condicionamentos e modelos – a dinâmica familiar apresenta influência no modo como o indivíduo irá se relacionar com o meio, inclusive em questões envolvendo atos ilícitos. No lar instalam-se as bases de crenças, valores e fundamentos de cada indivíduo, que se refletirão, mais tarde em condicionamentos positivos ou negativos em seus relacionamentos interpessoais.


III. A adolescência – alguns fatores contribuem para tornar o adolescente mais vulnerável à prática da delinquência, em comparação com o que acontece em outros momentos da vida. Três fatores atenuam essa vulnerabilidade aumentam a resiliência: 1) Vulnerabilidade do adolescente às mensagens que induzem à violência e à transgressão; 2) Complacência parental e 3) Percepção de falta de espaço no mundo adulto.

IV. Os microfatores externos. Se de um lado existe o indivíduo propenso à prática de delitos, por inúmeras razões, de outro, evidenciam-se indiscutíveis estímulos à delinquência: falta de limites durante a infância e também a expectativa de impunidade.


Está correto o que se afirma APENAS em:

Alternativas
Comentários
  • Acho que a II está correta também.

  • Gabarito D.

    Exatamente, Maria.

    O gabarito consta a II como correta também.


ID
832147
Banca
PUC-PR
Órgão
DPE-PR
Ano
2012
Provas
Disciplina
Psicologia
Assuntos

Pereira, Zanoni e Moser (2007) ao analisarem a formação do autoconceito e da autoestima em crianças institucionalizadas destacam:


I. A criança tratada com amor e respeito tende a internalizar esses sentimentos e sentir que é uma pessoa capaz de ser amada e respeitada. Ela cresce acreditando que é merecedora de amor e possui um espaço no mundo.


II. O autoconceito é nossa imagem de nós mesmos. É o que acreditamos em relação a quem somos – o quadro integral de nossos traços e capacidades, é um sistema de representação descritiva e avaliativa em torno do eu.


III. O autoconceito não é inato, mas é na infância que ele se estrutura/desenvolve, graças a influência de pessoas significativas do ambiente familiar, escolar e social.


IV. Autoestima é a avaliação que o indivíduo faz de si mesmo e que tende a se manter; expressa uma atitude de aprovação ou repulsa e até que ponto o sujeito se sente capaz, significativo, bem- sucedido e valioso. A autoestima é o juízo pessoal do valor que está expresso nas atitudes que um indivíduo tem para consigo mesmo.


Está correto o que se afirma APENAS em:


Alternativas
Comentários
  •  O autoconceito não é inato, mas é na infância que ele se estrutura/desenvolve (errado) - é durante toda a vida, graças a influência de pessoas significativas do ambiente familiar, escolar e social. ...

  • O autoconceito é nossa imagem de nós mesmos. É o que acreditamos em relação a quem somos o quadro integral de nossos traços e capacidades, é um sistema de representação descritiva e avaliativa em torno do eu.

    Isso ta certo?! Quem aqui, por mais tempo que tenha de análise, tem noção do quadro integral de traços e capacidades? 

  •  Autoestima é a avaliação que o indivíduo faz de si mesmo e que tende a se manter (...) - Isso está certo mesmo? A autoestima não varia?


ID
832159
Banca
PUC-PR
Órgão
DPE-PR
Ano
2012
Provas
Disciplina
Psicologia
Assuntos

A composição do papel do psicólogo no campo Jurídico foi objeto de análise no estudo realizado por Roehring e Siqueira (2007). Segundo as autoras, é correto afirmar em relação ao papel do psicólogo:


I. A construção da identidade profissional tem, nos conceitos de identidade, papel e relação, o suporte para a estruturação do papel profissional, num processo similar ao do desenvolvimento humano.


II. Nos elementos de configuração do papel do psicólogo jurídico, o termo jurídico refere-se a uma qualidade desta identidade – indica uma relação com o campo do Direito.


III. Ao propor um esquema da rede de sustentação da identidade profissional, as autoras propõem: Núcleo Identitário, referências representacionais, e sentimento de identidade profissional. As referências representacionais abordam o processo de trabalho: objeto, processo e produto.


IV. Cabe ao psicólogo visualizar, além dos fenômenos e processos psicológicos, o processo humano e subjetivo que se desenrola na vida dos indivíduos envolvidos, dinâmicas relacionais implicadas num conflito devem ser consideradas no momento das indicações e intervenções realizadas.


Está(ão) correta(s) APENAS:

Alternativas

ID
832186
Banca
PUC-PR
Órgão
DPE-PR
Ano
2012
Provas
Disciplina
Psicologia
Assuntos

Nos termos da legislação civil, a capacidade está relacionada à prática dos atos da vida civil, como, por exemplo, contrair matrimônio e administrar bens. O tema suscita diversas controvérsias em sede de ações judiciais, sendo invariavelmente necessário recorrer à perícia pelos profissionais de saúde, em especial da saúde mental. Nesse contexto, a incapacidade relativa refere-se a situações mais próximas DA NORMALIDADE, necessitando o indivíduo de assistência, por ter sua capacidade de discernimento reduzida, mas não abolida. Segundo o artigo 1.767 do Código Civil, estão sujeitos à Curatela:


I. Os excepcionais com completo desenvolvimento mental.


II. Os deficientes mentais, os ébrios habituais e os viciados em tóxicos.


III. Os pródigos.


IV. Aqueles que, por enfermidade ou deficiência mental, não tiverem o necessário discernimento para os atos da vida civil.


Está(ão) CORRETA(S):

Alternativas
Comentários
  • RESPOSTA: A lei mudou e por isso a questão se encontra desatualizada, neste caso, apenas o item III, os pródigos, se encontraria correto.

    O artigo 1.767 do Código Civil teve sua redação alterada pela lei 13.146/ 2015 passando a ter como redação: 

    Art. 1.767. Estão sujeitos a curatela:

    I - aqueles que, por causa transitória ou permanente, não puderem exprimir sua vontade;         

    III - os ébrios habituais e os viciados em tóxico;         

    V - os pródigos.

     


ID
854032
Banca
FCC
Órgão
TJ-PE
Ano
2007
Provas
Disciplina
Psicologia
Assuntos

A Psicologia Jurídica é uma especialidade da Psicologia que tem sua interface com o Direito. Os setores da Psicologia Jurídica são diversos. Segundo Fátima França, há os setores tradicionais, como a atuação em Fóruns e Prisões, e há também atuações inovadoras como a

Alternativas
Comentários
  • Reflexões sobre psicologia jurídica e seu panorama no Brasil
    http://pepsic.bvsalud.org/scielo.php?pid=S1516-36872004000100006&script=sci_arttext


    A Psicologia Jurídica é uma emergente área de especialidade da ciência psicológica, se comparada às áreas tradicionais de formação e atuação da Psicologia como a Escolar, a Organizacional e a Clínica. É próprio desta especialidade sua interface com o Direito, com o mundo jurídico, resultando encontros e desencontros epistemológicos e conceituais que permeiam a atuação do psicólogo jurídico. Os setores da Psicologia Jurídica são diversos. Há os mais tradicionais, como a atuação em Fóruns e Prisões, e há também atuações inovadoras como a Mediação e a Autópsia psíquica, uma avaliação retrospectiva mediante informações de terceiros. O presente trabalho focaliza a Psicologia Jurídica Brasileira. Objetiva apresentar e discutir a definição de Psicologia Jurídica e sua relação com o Direito, destacar seus setores de atuação de acordo com os trabalhos apresentados no III Congresso Ibero- Americano de Psicologia Jurídica realizado no Brasil em 1999 e abordar os desafios para a Psicologia Jurídica Brasileira.
  • A autópsia psicológica é uma estratégia utilizada para delinear as características psicológicas de vítimas de morte violenta, sendo utilizada no curso de uma investigação de morte para auxiliar a determinar o modo de morte de um indivíduo, especialmente em casos duvidosos.

     

    http://www.scielo.br/pdf/csc/v17n8/03.pdf


ID
886060
Banca
VUNESP
Órgão
TJ-SP
Ano
2012
Provas
Disciplina
Psicologia
Assuntos

Dentre os procedimentos técnicos envolvidos no trabalho do psicólogo judiciário, Shine (2003) destaca a importância da leitura dos autos. Para ele, esse procedimento consiste em

Alternativas
Comentários
  • b) procurar subsídios para a estratégia de avaliação

  • Nesta questão até sem mesmo ler o livro completo de Shine podemos concluir que em todo o caso quando solicitado a realização de uma avaliação psicológica o profissional da psicologia deve buscar subsídios para o planejamento do processo de avaliação tendo como foco de trabalho o objetivo deste ser solicitado e o conhecimento funcionamento psíquico do sujeito como um todo.

  • De acordo com Shine:

     

    Recomenda-se que, quando do recebimento do encaminhamento, proceda-se o recebimento, leitura e resumo dos autos. Concluindo em relação à leitura dos autos, destaca-se uma finalidade pragmática que não varia muito dentre os trabalhos: procura-se subsídios para traçar uma estratégia de avaliação.

    Shine complementa que um dos erros na perícia é a escolha errada das técnicas, lendo os autos poderá ajudar no planejamento da avaliação.

    Amigos, bons estudos!


ID
917434
Banca
UEG
Órgão
PM-GO
Ano
2013
Provas
Disciplina
Psicologia
Assuntos

Com base nas resoluções do Conselho Federal de Psicologia, qual das alternativas abaixo está em contradição com as normas legais brasileiras?

Alternativas
Comentários
  • Gabarito: d) O psicólogo assistente técnico e o psicólogo perito devem utilizar os mesmos instrumentos durante a avaliação psicológica e, consequentemente, diminuir os prejuízos na qualidade da perícia realizada.


ID
917437
Banca
UEG
Órgão
PM-GO
Ano
2013
Provas
Disciplina
Psicologia
Assuntos

Autores brasileiros da Psicologia Jurídica (Angelim e Ribeiro, 2012) destacam mudanças recentes nas práticas jurídicas que apontam para novas formas de atuação dos psicólogos nessa área. O mais importante dos novos desafios é o de teorizar sobre a importância do Estado para legitimar e proteger a diversidade humana. As mudanças recentes nas práticas jurídicas destacadas são as leis que tratam

Alternativas

ID
926815
Banca
CESPE / CEBRASPE
Órgão
CNJ
Ano
2013
Provas
Disciplina
Psicologia
Assuntos

De acordo com o Código Civil em vigor, julgue os itens
subsequentes, relativos a aspectos gerais relativos à psicopatologia.

Os doentes mentais, os viciados em tóxicos e os excepcionais com incompleto desenvolvimento mental estão sujeitos à curatela.

Alternativas
Comentários
  • CC - Lei nº 10.406 de 10 de Janeiro de 2002

    Institui o Código Civil .

     

    Art. 1.767. Estão sujeitos a curatela:

     

    - aqueles que, por enfermidade ou deficiência mental, não tiverem o necessário discernimento para os atos da vida civil;

     

    II - aqueles que, por outra causa duradoura, não puderem exprimir a sua vontade;

     

    III - os deficientes mentais, os ébrios habituais e os viciados em tóxicos;

     

    IV - os excepcionais sem completo desenvolvimento mental;

     

    - os pródigos.

  • A título de informação, curatela é um instituto jurídico pelo qual o curador tem o encargo imposto pelo juíz de cuidar dos interesses de outrem que se encontra incapaz de fazê-lo. A nomeação do curador é feita pelo juiz, que estabelece, conforme previsão legal, as atribuições desse curador.

  • Atualização!!!

    Art. 1.767. Estão sujeitos a curatela:

    I - aqueles que, por causa transitória ou permanente, não puderem exprimir sua vontade;    

    II - (Revogado);

    III - os ébrios habituais e os viciados em tóxico;        

    IV - (Revogado);

    V - os pródigos.


ID
926818
Banca
CESPE / CEBRASPE
Órgão
CNJ
Ano
2013
Provas
Disciplina
Psicologia
Assuntos

De acordo com o Código Civil em vigor, julgue os itens
subsequentes, relativos a aspectos gerais relativos à psicopatologia.

São absolutamente incapazes de exercer pessoalmente os atos da vida civil os menores de dezoito anos de idade, os ébrios habituais e os que, por deficiência mental ou enfermidade, não tiverem consciência da prática de seus atos, ou mesmo aqueles que não puderem exprimir sua vontade.

Alternativas
Comentários
  • ERRADO. O ERRO ESTÁ NOS ÉBRIOS E NOS MENORES DE 18 ANOS POIS ESTES SÃO RELATIVAMENTE INCAPAZES.
    Art. 3o São absolutamente incapazes de exercer pessoalmente os atos da vida civil:
    I - os menores de dezesseis anos;
    II - os que, por enfermidade ou deficiência mental, não tiverem o necessário discernimento para a prática desses atos;
    III - os que, mesmo por causa transitória, não puderem exprimir sua vontade.

    Art. 4o São incapazes, relativamente a certos atos, ou à maneira de os exercer:
    I - os maiores de dezesseis e menores de dezoito anos;
    II - os ébrios habituais, os viciados em tóxicos, e os que, por deficiência mental, tenham o discernimento reduzido;
    III - os excepcionais, sem desenvolvimento mental completo;
    IV - os pródigos.
  • A título de atualização:

    Lei nº 10.406/2002 - Código Civil.

    Art. 3São absolutamente incapazes de exercer pessoalmente os atos da vida civil os menores de 16 (dezesseis) anos. 

    I - REVOGADO; 

    II - REVOGADO; 

    III - REVOGADO. 

    Art. 4º São incapazes, relativamente a certos atos ou à maneira de os exercer: 

    I - os maiores de dezesseis e menores de dezoito anos;

    II - os ébrios habituais e os viciados em tóxico; 

    III - aqueles que, por causa transitória ou permanente, não puderem exprimir sua vontade; 

    IV - os pródigos.


ID
926821
Banca
CESPE / CEBRASPE
Órgão
CNJ
Ano
2013
Provas
Disciplina
Psicologia
Assuntos

De acordo com o Código Civil em vigor, julgue os itens
subsequentes, relativos a aspectos gerais relativos à psicopatologia.

Em caso de incapacidade relativa, que ocorre quando a capacidade civil de fato está próxima da normalidade, os curadores são desnecessários.

Alternativas
Comentários
  • ERRADA

    Incapacidade Absoluta
    :
    o absolutamente incapaz é representado, e o ato que praticar sozinho será considerado nulo (nulidade absoluta).
    São absolutamente incapazes:
    a) Menores de 16 anos.
    b) Os que por enfermidade ou doença mental não tenham discernimento.
    c) Aqueles que por causa transitória não possam expressar sua vontade (coma, hipnotizado).
    d) O ausente quanto aos bens deixados sob a administração do curador.

    Incapacidade Relativa: o relativamente incapaz é assistido (ajudado), o ato que praticar sozinho será anulável (nulidade relativa ou anulabilidade).
    São relativamente incapazes:
    a) Os menores entre 16 e 18 anos.
    b) O ébrio habitual.
    c) O viciado em tóxicos.
    d) Os deficientes mentais que tiverem o discernimento reduzido.
    e) O excepcional sem desenvolvimento mental completo.
    f) O pródigo (aquele que gasta imoderadamente seu patrimônio)

    Fonte: http://www.macetesjuridicos.com.br/2009/09/incapaciade-absoluta-e-relaiva.html
  • RESPOSTA: ERRADO

    ATENÇÃO!  O Estatuto da Pessoa com Deficiência, instituído pela Lei 13.146/15, que entrou em vigor no dia 2 de janeiro de 2016, modificou dispositivos do Código Civil que tratavam da capacidade civil. Seus artigos 114 e 123, inciso II revogaram os incisos do artigo 3º do CC e alteraram seu caput, como também modificaram os incisos II e III do artigo 4º do CC.

    Agora, apenas as pessoas menores de 16 anos são absolutamente incapazes, sendo considerados relativamente incapazes as pessoas entre 16 e 18 anos, os pródigos, os ébrios habituais e os viciados em tóxicos e aqueles que, por causa transitória ou permanente, não puderem exprimir sua vontade.

    ABSOLUTAMENTE INCAPAZ: 

    Art. 3o  São absolutamente incapazes de exercer pessoalmente os atos da vida civil os menores de 16 (dezesseis) anos. 

    RELATIVAMENTE INCAPAZ:

    Art. 4o  São incapazes, relativamente a certos atos ou à maneira de os exercer:             

    I - os maiores de dezesseis e menores de dezoito anos;

     II - os ébrios habituais e os viciados em tóxico;        

     III - aqueles que, por causa transitória ou permanente, não puderem exprimir sua vontade;        

    IV - os pródigos.

    https://www.planalto.gov.br/ccivil_03/leis/2002/L10406.htm

    Art. 1.767. Estão sujeitos a curatela:

    I - aqueles que, por causa transitória ou permanente, não puderem exprimir sua vontade;         

    III - os ébrios habituais e os viciados em tóxico;         

    V - os pródigos.

     

    https://www.planalto.gov.br/ccivil_03/leis/2002/L10406.htm

     

     

  • "incapacidade relativa se aplica, sim, aos menores e maiores de Idade. Ocorre que, como a capacidade é a regra, presumida até que se prove o contrário, a incapacidade relativa deve ser decretada pelo juiz. O Juiz nomeará um curador para o relativamente incapaz, delimitando os limites da curatela"


ID
926824
Banca
CESPE / CEBRASPE
Órgão
CNJ
Ano
2013
Provas
Disciplina
Psicologia
Assuntos

De acordo com o Código Civil em vigor, julgue os itens
subsequentes, relativos a aspectos gerais relativos à psicopatologia.

Como o comportamento pródigo pode ser constatado em pessoas com determinados transtornos mentais, é necessária a avaliação criteriosa das pessoas que apresentam esse tipo de comportamento, para que seja excluída a hipótese de doença psiquiátrica de base.

Alternativas
Comentários
  • "Nesse sentido, pródigo é considerado a pessoa que dissipa de maneira desordenada, através de gastos excessivos e sem fins úteis, seu patrimônio e suas rendas."

    Fonte: https://jus.com.br/artigos/4863/contribuicao-ao-estudo-da-prodigalidade


ID
926842
Banca
CESPE / CEBRASPE
Órgão
CNJ
Ano
2013
Provas
Disciplina
Psicologia
Assuntos

No que se refere à atuação do psicólogo e às técnicas e teorias
utilizadas por esse profissional, julgue os itens subsecutivos.

No caso de o chefe de uma seção encaminhar um seu subordinado ao serviço de apoio psicossocial, após esse subordinado se envolver em diversos problemas com colegas de trabalho, caberá ao psicólogo responsável pelo seu atendimento intervir com o objetivo de promover o convívio saudável do grupo e propor medidas sociais preventivas para evitar a reincidência.

Alternativas
Comentários
  • No caso de o chefe de uma seção encaminhar um seu subordinado ao serviço de apoio psicossocial - intervenção não é de psicoterapia, mas de ações de promoção de saúde e desenvolvimento social= 

    promover o convívio saudável do grupo e propor medidas sociais preventivas para evitar a reincidência.


ID
926848
Banca
CESPE / CEBRASPE
Órgão
CNJ
Ano
2013
Provas
Disciplina
Psicologia
Assuntos

No que se refere à atuação do psicólogo e às técnicas e teorias
utilizadas por esse profissional, julgue os itens subsecutivos.

Forclusão é um termo do meio jurídico utilizado pela psicanálise para se referir a determinado processo psíquico que, como proposta pelo CID-10 ou DSM-IVTR, deixou de ocorrer em dado momento do desenvolvimento, mas que pode advir, posteriormente, sem resultar em danos psíquicos, até os sete anos.

Alternativas
Comentários

  • O erro deve estar em TERMO DO MEIO JURIDICO e todo o resto do enunciado, com o se vê abaixo, ao contrário é um termo tipicamente psicanalítico.
    Segundo Freud a Verwerfung designa a expulsão de um conteúdo da experiência para fora do eu, em função do princípio do prazer. A existência na realidade encontrar-se-ia denegada a sua representação. A Verwerfung implica a negação da identificação, na medida em que esta assenta na assunção do patronímio, tal como é explicitado por Freud em O Homem Moisés e a Religião Monotheísta.

    Lacan introduz o conceito forclusion para designar a falta que dá à psicose a sua condição essencial, com a estrutura que a separa das neuroses. Trata-se de um significante, o Nome-do­-Pai, que foi rejeitado da ordem simbólica da linguagem, reaparecendo no real, por exemplo, sob forma de alucinação. Este significante tem como correlativo o significado da castração. 0 sujeito psicótico, não tendo passado pela castração simbólica, vê-se, em determinadas condições, confrontado com a castração real.

    http://www.edtl.com.pt/?option=com_mtree&task=viewlink&link_id=210&Itemid=2

  • No Seminário 5 - As Formações do Inconsciente, Lacan (1957-1958/1999) formaliza o conceito de foraclusão do Nome-do-Pai, tal como já vinha desenvolvendo desde o Seminário 3 - As Psicoses (Lacan, 1955-1956/1992) e no texto "De uma Questão Preliminar a Todo Tratamento Possível da Psicose" (Lacan, 1958/1998), quando foi assim nomeado.

    Nesse último texto, define a foraclusão como o mecanismo que estaria na origem da estrutura psicótica - estabelecendo, aí, uma clínica diferencial em relação à neurose, cujo mecanismo fundante é o recalque - e que consistiria na rejeição do significante do Nome-do-Pai para fora do registro do simbólico, sendo esse fracasso da metáfora paterna, essa falha na operação de castração, o que conferiria à psicose sua condição essencial. 

    A verwerfung original será tida por nós, portanto, como foraclusão do significante. No ponto em que, veremos de que maneira, é chamado o Nome-do-Pai, pode pois responder no Outro um puro e simples furo, o qual pela carência de efeito metafórico, provocará um furo correspondente na significação fálica. (Lacan, 1958/1998, p. 564) 

    Os significantes foracluídos, diferentemente do que ocorre no recalque, no qual são reintegrados ao inconsciente via simbólico, retornam de fora pela via do real, como é o caso dos fenômenos alucinatórios. 

    No Seminário 5, Lacan estabelece o significante do Nome-do-Pai como aquele que fundamenta a Lei, que representa o Outro do Outro. O Outro é entendido como tesouro significante e garantido pela Lei para exercer sua função. Trata-se, portanto, de um Outro completo e consistente. Dito de outra forma, a função do pai é central na questão do Édipo; o pai, aqui, não só é visto como pai simbólico, mas mais precisamente como metáfora, cuja função no complexo de Édipo é substituir o primeiro significante (o materno) introduzido na simbolização.

    É o que propõe, já no Seminário 3

    O complexo de Édipo quer dizer que a relação imaginária, conflituosa, incestuosa nela mesma, está destinada ao conflito e à ruína (…) é preciso aí uma lei, uma cadeia, uma ordem simbólica, uma intervenção da ordem da palavra, isto é, do pai. Não o pai natural, mas do que se chama o pai. A ordem que impede a colisão e o rebentar da situação no conjunto está fundada na existência desse nome do pai. (…) Essa Lei fundamental é simplesmente uma Lei de simbolização. É o que o Édipo quer dizer. (Lacan, 1955-1956/1992, pp. 100-114)

    Considerando-se a psicose como portadora de uma falha simbólica estrutural, a noção de suplência, nesse momento, pode ser entendida como algo que metaforiza a função paterna foracluída, como é o caso da metáfora delirante. Apesar de não se referir à escrita de Schreber, formalmente, como exercendo função de suplência, refere-se a sua língua fundamental como uma rede de natureza simbólica que impediria a dissolução total de seu imaginário.

  • Q viagem!!


ID
926863
Banca
CESPE / CEBRASPE
Órgão
CNJ
Ano
2013
Provas
Disciplina
Psicologia
Assuntos

Em relação às atividades do psicólogo no âmbito jurídico, julgue os
itens que se seguem, com base na legislação pertinente.

A leitura dos autos e as entrevistas realizadas com o indivíduo que cometeu o delito norteiam a seleção dos exames e das baterias de testes que podem confirmar ou refutar as hipóteses diagnósticas levantadas.

Alternativas
Comentários
  • Acertei, mas fiquei na dúvida em de repente as hipóteses diagnósticas também serem norteadores na escolha dos exames e das baterias. Alguém sabe informar sobre isso?

  • fiquei na dúvida também pelo fato dos exames serem escolhidos com base nas capacidades do profissional, já errei uma questão que seguia essa outra ideia contrária
  • Pessoal nós que somos da área da Psicologia precisamos cobrar do QC explicações dos professores. É um cúmulo vermos professores comentando o gabarito em praticamente em todas as disciplinas e em Psicologia essa escassez. Avaliação Psicológica é muito cobrada nos editais e precisamos desse suporte aqui também. SOLICITEM O GABARITO COMENTADO DO PROFESSOR!


ID
926875
Banca
CESPE / CEBRASPE
Órgão
CNJ
Ano
2013
Provas
Disciplina
Psicologia
Assuntos

Em relação às atividades do psicólogo no âmbito jurídico, julgue os
itens que se seguem, com base na legislação pertinente.

Em casos de ação civil, até três psiquiatras podem atuar como peritos, sendo um deles nomeado pelo juiz.

Alternativas

ID
1068973
Banca
FAPERP
Órgão
TJ-PB
Ano
2012
Provas
Disciplina
Psicologia
Assuntos

Historicamente os maiores clientes do psicólogo jurídico são:

Alternativas
Comentários
  • GABARITO:C

    crianças, adolescentes e doentes mentais.


ID
1068976
Banca
FAPERP
Órgão
TJ-PB
Ano
2012
Provas
Disciplina
Psicologia
Assuntos

Em um processo de interdição, o psicólogo pode ser solicitado para emitir laudo psicológico do interditando a fim de assistir na decisão judicial de incapacidade civil. Podem ser considerados incapazes:

Alternativas
Comentários
  • GABARITO: D

     


ID
1068979
Banca
FAPERP
Órgão
TJ-PB
Ano
2012
Provas
Disciplina
Psicologia
Assuntos

Quando solicitado a realizar uma avaliação psicológica no âmbito jurídico, o psicólogo deve:

Alternativas
Comentários
  • Na avaliação psicológica o psicólogo (a) deve estar sempre atendo a solicitação a qual a mesma foi feita para não utilizar instrumentos e técnicas inadequadas ao caso em questão.

  • Utilizar instrumentos que irão embasar à pratica sob domínio: comportamental, afetivo e cognitivo. Um bom psicodiagnóstico de inicio pode ser extremamente importante. 

  • GABARITO: A

    escolher os instrumentos e as técnicas adequadas ao fenômeno psicológico que se propõem a investigar

  • Os instrumentos devem ser escolhidos baseados nos propósitos e contextos para os quais é indicado. Cabe ao psicólogo e somente a este escolher os instrumentos com os quais irá trabalhar. A subordinação do perito ao juiz é apenas administrativa, nunca técnica.

    Quem escolheu a busca não pode recusar a travessia - Guimarães Rosa

    Gabarito: A

  • Procurei, mas não encontrei.


ID
1068994
Banca
FAPERP
Órgão
TJ-PB
Ano
2012
Provas
Disciplina
Psicologia
Assuntos

Considera-se inscrito no campo da Psicologia Jurídica o profissional de psicologia que:

Alternativas
Comentários
  • GABARITO : D

    Todas as alternativas estão corretas.

  • A fundamentação para esta questão está no documento do Crepop "Referências Técnicas para a Atuação do Psicólogo em Varas de Família". Vejamos:

    "Cabe esclarecer, portanto, que, na designação psicólogos que atuam em Varas de Família, estão sendo considerados profissionais lotados em Varas de Família, assim como aqueles que não possuem vínculo empregatício no Poder Judiciário, mas são indicados por juízes para ser peritos, bem como os contratados por uma das partes como assistentes técnicos. Incluem-se também nessa designação os que são cedidos por órgãos públicos para desempenhar atividade profissional perante as Varas de Família, bem como aqueles lotados em outras instituições, mas que produzem trabalhos endereçados ao Juízo de Família. Salienta-se que os psicólogos jurídicos que integram equipes interprofissionais nos Tribunais de Justiça podem ser lotados nas Varas da Infância e Juventude, atendendo separada ou cumulativamente as Varas de Família" (p. 13).

    Quem escolheu a busca não pode recusar a travessia - Guimarães Rosa

    Gabarito: D


ID
1068997
Banca
FAPERP
Órgão
TJ-PB
Ano
2012
Provas
Disciplina
Psicologia
Assuntos

São setores mais recentes da Psicologia Jurídica:

Alternativas
Comentários
  • Humilhou hum
  • É nooooooox
  • d) CORRETA.

    Nos casos de Separação e Divórcio, por exemplo, o Psicólogo pode atuar na Mediação de modo a tentar encontrar um acordo entre as partes, pois são processos que envolvem partilha de bens, guarda de filhos, estabelecimento de pensão alimentícia e direito à visitação.

     

    No caso da Vitimologia ele avalia o comportamento e a personalidade da vítima, traçando o perfil e compreendendo as reações das vítimas perante a infração penal, onde se busca averiguar se a prática do crime foi estimulada pela atitude da vítima, o que pode denotar uma cumplicidade passiva ou ativa para o criminoso. Portanto, a análise é feita desde a ocorrência até as consequências do crime. A Vitimologia também se dedica à aplicação de medidas preventivas e à prestação de assistência às vítimas, visando à reparação de danos causado pelo delito.

  • mediação e vitimologia.


ID
1069030
Banca
FAPERP
Órgão
TJ-PB
Ano
2012
Provas
Disciplina
Psicologia
Assuntos

A posição que o Conselho Federal de Psicologia defende acerca da proposta de redução da maioridade penal no Brasil é fundamentada por 10 razões, entre as quais encontra-se:

Alternativas

ID
1085401
Banca
CESPE / CEBRASPE
Órgão
MPE-AC
Ano
2014
Provas
Disciplina
Psicologia
Assuntos

Considerando a importância das técnicas psicológicas de inquirição de testemunhas para o bom desempenho das funções jurisdicionais, assinale a opção correta.

Alternativas
Comentários
  • Aspectos concernentes à individualidade e experiências pessoais podem interferir no relato testemunhal do fato.

    GABARITO: E


ID
1177093
Banca
CESPE / CEBRASPE
Órgão
TC-DF
Ano
2014
Provas
Disciplina
Psicologia
Assuntos

Acerca da intervenção do psicólogo, julgue os itens subsequentes.

Compete ao psicólogo forense o diagnóstico e tratamento de sintomatologias e patologias de forma a reduzir o sofrimento do indivíduo e diminuir as ocorrências de delitos.

Alternativas
Comentários
  • Psicologo forense não trata

  • Psicologia Forense é o estudo do comportamento desenvolvido dentro de ambientes regulados juridicamente, assim como da evolução dessas regulamentações jurídicas e de como os grupos sociais desenvolvem-se nesse processo. (CLEMENTE, 1998). Essa é uma área da Psicologia aplicada que busca promover um melhor exercício do Direito. As áreas de atuação do psicólogo forense (MIRA Y LÓPEZ, 2005):

    1. Psicologia do testemunho;
    2. Obtenção de evidência delituosa (confissão com provas);
    3. Compreensão do delito (motivação psicológica);
    4. Informação forense a seu respeito;
    5. Reforma moral do delinquente;
    6. Higiene mental (como evitar que ocorram conflitos com as leis?). 

    O tema da “higiene mental” é muito abordado e pesquisado pelos psicólogos jurídicos dos Estados Unidos, Alemanha, Áustria e Inglaterra. Esse ramo da Psicologia é a ciência que trata da aplicação de todos os ramos e saberes da Psicologia para responder as perguntas da Justiça e colaborar com a sua administração, promovendo a melhora do exercício do Direito (URRA, 1993). 

    Com isso, o psicólogo forense busca compreender o humano a partir dos princípios da:

    1. Ênfase na análise individual relacionada com o seu contexto social, político, econômico;
    2. Ideia de que os comportamentos devem ser analisados em todos os âmbitos, não só no aspecto criminal, mas também no ambiental e emocional;
    3. Crença na ideia de que o ser humano orienta-se por sua “escala de necessidades”, que vão desde a subsistência à dimensão moral, religiosa, etc.; 
    4. Avaliação da motivação psicológica e de como os estímulos do ambiente são processados e interpretados e de como adquirem significado pessoal.

    A Psicologia Forense é uma das atividades do psicólogo, que é relativa à descrição dos processos mentais e comportamentais, conforme o uso de técnicas psicológicas reconhecidas, respondendo estritamente à demanda judicial, sem emitir juízo de valor. (SILVA, E. L., 2009). 

    Nesse sentido, vale a lembrança de que o psicólogo responde judicialmente pelos efeitos e resultados da medida judicial pautada pelo seu trabalho. (SHINE, 2008).

    Essa área da Psicologia nasce em um espaço no qual o psicólogo coloca os seus conhecimentos a serviço do juiz (o julgador), trazendo a este âmbito a dimensão da realidade psicológica dos agentes envolvidos. (SILVA, D. M. P., 2009). 

    A Psicologia Forense situa-se na confluência de vários saberes. (FONSECA, 2006). Há inúmeras conexões, como, por exemplo, com o Direito, a Psiquiatria, a Medicina, o Serviço Social, a Sociologia, a Antropologia, várias áreas da própria Psicologia (ex.: desenvolvimento, experimental, cognitiva, clínica, avaliação, psicometria), entre outras. Assim, a multiplicidade de saberes e de competências é uma das marcas da Psicologia Forense.

    Fonte: PORTAL EDUCAÇÃO - Cursos Online : Mais de 1000 cursos online com certificado 
    http://www.portaleducacao.com.br/psicologia/artigos/35180/definicao-de-psicologia-forense#ixzz4G52b9kOb

  • Não cabe ao psicólogo forense o tratamento de eventuais patologias. O que está ao seu alcance é a sugestão do acompanhamento psicológico e o encaminhamento para a rede de saúde mental do território.


ID
1265806
Banca
TJ-PR
Órgão
TJ-PR
Ano
2013
Provas
Disciplina
Psicologia
Assuntos

É interessante que possamos, como profissionais da área psi, quer seja como psicólogos do Judiciário ou pesquisadores, nos interrogar sobre nossas práticas, recusando-nos a assumir apenas o lugar de um técnico, de um agente solucionador de problemas imediatos. É importante que possamos nos colocar como problematizadores das articulações coletivas que contemplem as diferentes instituições que atravessam o tecido social, o Judiciário, ou a nós próprios. Ou seja, que possamos entender a Psicologia como uma prática, uma ferramenta de intervenção social e nós, como sujeitos comprometidos.

                                                                                                      (Coimbra; Ayres e Nascimento, 2008)

Assinale a alternativa que vai ao encontro do que propõe o trecho citado:

Alternativas

ID
1265824
Banca
TJ-PR
Órgão
TJ-PR
Ano
2013
Provas
Disciplina
Psicologia
Assuntos

De acordo com Araújo (2004), na vida das vítimas, as consequências do crime do abuso sexual podem surgir de formas diferentes, dependendo do número de agressões sofridas, do período de tais violências e do tipo de indução ao ato. A reação dos adultos e/ou familiares a essa situação também pode ser significativa. Nesse sentido, identifique as afirmativas a seguir como verdadeiras (V) ou falsas (F):

( ) É preciso acolher o sujeito.
( ) É necessário não intensificar o seu sofrimento.
( ) É importante acreditar na capacidade dessa criança ou adolescente de ter menos sequelas emocionais.
( ) É preciso oferecer apoio ao sujeito.

Assinale a alternativa que apresenta a sequência correta, de cima para baixo.

Alternativas

ID
1265833
Banca
TJ-PR
Órgão
TJ-PR
Ano
2013
Provas
Disciplina
Psicologia
Assuntos

Considere os seguintes problemas:

1. A miséria social.
2. A carência de apoio socioeducativo.
3. A ausência de prevenção em relação à violência doméstica.
4. A ausência de qualquer tipo de reabilitação para as famílias de origem.

A causa do problema de crianças e adolescentes abandonados deve-se a:

Alternativas
Comentários
  • Na minha opinião, questão mal-elaborada.

    Penso que o correto seria falar das fragilidades, e não ausência, em relação à prevenção da violência e reabilitação para as famílias de origem.

    A exemplo, podemos citar a atuação dos CRAS e CREAS - Centro de Referência Especializados de Assitência Social que atuam na prevenção, função protetiva das famílias e em situação de violação de direitos, respaldados na Tipioficação Nacional de Serviços Socioassistenciais e no SUAS.


ID
1265836
Banca
TJ-PR
Órgão
TJ-PR
Ano
2013
Provas
Disciplina
Psicologia
Assuntos

A expressão “abuso sexual” não faz parte das definições de crimes de natureza sexual do Código Penal, em vigor desde 1940. É preciso encaixar o termo dentro das tipificações existentes no Código, o que pode dificultar ainda mais a avaliação final do juiz. Sobre o assunto, considere as seguintes categorias:

1. Estupro (art. 213).
2. Atentado violento ao pudor (art. 214).
3. Presunção da violência (art. 224).

O abuso sexual é julgado dentro das seguintes categorias legais:

Alternativas

ID
1265842
Banca
TJ-PR
Órgão
TJ-PR
Ano
2013
Provas
Disciplina
Psicologia
Assuntos

Sobre o Conselho Tutelar, previsto no art. 131 do ECA, considere as seguintes afirmativas:

1. É permanente e autônomo.
2. Não é jurisdicional.
3. É encarregado de zelar pelo cumprimento dos direitos da criança e do adolescente.

Assinale a alternativa correta.

Alternativas
Comentários
  • Art. 131 - O Conselho Tutelar é  órgão permanente e autônomo, não jurisdicional, encarregado pela sociedade de zelar pelo cumprimento dos direitos da criança e do adolescente, definidos nesta Lei. 

  • LEI Nº 8.069/1990

     

    Art. 131 – O Conselho Tutelar é órgão permanente e autônomo (1), não jurisdicional (2), encarregado pela sociedade de zelar pelo cumprimento dos direitos da criança e do adolescente (3), definidos nesta Lei;

     

    Quem escolheu a busca não pode recusar a travessia - Guimarães Rosa

    ------------------- 

    Gabarito: C


ID
1265845
Banca
TJ-PR
Órgão
TJ-PR
Ano
2013
Provas
Disciplina
Psicologia
Assuntos

Considere as seguintes técnicas de depoimento:

1. Depoimento sem dano.
2. Audiência sem trauma.
3. Audiência interprofissional protegida à vítima.
4. Atendimento não revitimizante de crianças e adolescentes vítimas de violência.

São técnicas para obtenção dos depoimentos especiais:

Alternativas
Comentários
  • Depoimento sem dano: Escuta em ambiente especial, uma sala separada, onde ficam o Psicólogo, criança (vítima) e o Juíz, Advogados, acusado..em outra sala.

    Audiência sem trauma: A vítima é ouvida pelo juiz, promotor e advogados em um ambiente diferente, sem a presença do agressor, e acompanhada por um psicólogo..

    Audiência interprofissional protegida à vítima: É o trabalho com vários profissionais, Psicólogo, Ass Social...que em conjunto irão proteger a privacidade, sentimentos, medos.... da criança / adolescente

    Atendimento não revitimizante de crianças e adolescentes vítimas de violência: é a implantação da escuta especial, procedimento que evita que crianças e adolescentes vítimas de violência (especialmente de violência sexual) tenham que contar sua história repetidas vezes durante o itinerário que percorrem ao serem atendidas, em função da ocorrência, pelos sistemas de saúde, segurança, serviço social e justiça. 


ID
1265863
Banca
TJ-PR
Órgão
TJ-PR
Ano
2013
Provas
Disciplina
Psicologia
Assuntos

A autópsia psicológica é um procedimento complexo que tem como finalidade reconstruir a bibliografia da pessoa falecida (história clínica completa) por meio de entrevista com terceiros, comumente chamados de informantes (cônjuge, filhos, pais, amigos, professores, médicos etc.).

                                                                                          (Asnis, Macedo e Werlang, 2005, p. 193)

A utilização desse procedimento:

Alternativas

ID
1265869
Banca
TJ-PR
Órgão
TJ-PR
Ano
2013
Provas
Disciplina
Psicologia
Assuntos

A entrevista retrospectiva na autópsia psicológica pode:

1. Determinar as características psicológicas do falecido.
2. Distinguir o ato do sujeito entre uma situação de homicídio ou acidente.
3. Favorecer a identificação de situações de risco sociodemográfico do suicídio.
4. Dificultar aos familiares do suicida o enfrentamento de uma fase de grande estabilidade emocional.

Estão corretos os itens:

Alternativas
Comentários
  • A autopsia psicológica nasce como um procedimento para assessorar médicos forenses para classificar com maior precisão o registro de suicídio (ato de se matar intencionalmente) no certificado de óbito. Rapidamente torna-se um procedimento aceito e muito utilizado.Então, a autópsia psicológica é uma estratégia utilizada para delinear as características psicológicas de vítimas de morte violenta, sendo utilizada durante o curso de uma investigação de morte, para auxiliar a determinar o modo de morte de um indivíduo, especialmente em casos duvidosos. Com o passar do tempo, este recurso que muito auxiliou a médicos legistas e profissionais da área do direito penal e cível passou, também, a contribuir na corroboração e/ou identificação de novos fatores de risco e correlatos sociodemográficos do suicídio.

    A estratégia de autópsia psicológica possibilita reconstruir a biografia da pessoa falecida focando a intenção do morto em relação a sua própria morte. Nos casos apresentados é clara a vivência de uma dor psicológica insuportável, o que possibilita afirmar que o suicídio, não é um ato aleatório, sem finalidade, pelo contrário, é vivenciado como a melhor saída disponível frente a uma situação conflituosa para o sujeito e tem o propósito de viabilizar uma solução para um sofrimento muito intenso. O alvo é interromper o fluxo doloroso de consciência, parando com o sentimento invasor de desesperança que deixa o indivíduo derrotado e sem saída para a vida.

    Mas, não podemos perder de vista que o suicídio, em qualquer faixa etária, pode ser prevenido.


  • Eita! nunca ouvi dalar disso :O . Tem fonte ? Obrigado pelas informações.

  • o 4 é muito sem noção.

    Mayra, um livro fácil de encontrar e que explana o assunto de forma geral é o Psicodiagnóstico V (2003), da Alcides Cunha.


ID
1265887
Banca
TJ-PR
Órgão
TJ-PR
Ano
2013
Provas
Disciplina
Psicologia
Assuntos

Os autores variam quanto à estimativa do ato de simulação e do engano no contexto forense. De acordo com Vargas (1990), existem diferentes tipos de simulação. Sobre o tema, considere os seguintes itens:

1. Simulação fantasiosa.
2. Pré-simulação anterior.
3. Parassimulação, supersimulação ou simulação aumentada.
4. Metassimulação ou simulação residual.

São considerados tipos de simulação:

Alternativas
Comentários
  • 3 tipos de simulação descritos por Vargas:

    Pré-simulação ou simulação anterior: o simulador planeja e executa seus sintomas com antecedência, criando um reconhecimento social de sua doença.

    Parassimulação, supersimulação ou simulação aumentada: a pessoa copia e imita sintomas e condutas de outras pessoas, com o objetivo de obter vantagens.

    Metassimulação ou simulação residual: forma mais frequente, que é quando o sujeito, já recuperado, continua a fingir os sintomas.


ID
1265893
Banca
TJ-PR
Órgão
TJ-PR
Ano
2013
Provas
Disciplina
Psicologia
Assuntos

De acordo com Marshall (2006), existem algumas áreas-alvo para a avaliação de agressores sexuais. Assinale a alternativa que identifica essas áreas.

Alternativas
Comentários
  • A resposta está na página 147 do livro "Psicologia Forense: Pesquisa, Prática Clínica e Aplicações" Matthew T. Huss

    http://books.google.com.br/books?id=lvvzlEYoNvAC&pg=PA147&lpg=PA147&dq=De+acordo+com+Marshall,+existem+algumas+áreas-alvo+para+a+avaliação+de+agressores+sexuais.&source=bl&ots=Plx--IRnJf&sig=BDo6y2-KM2jfL6M_gxg9qyFniM0&hl=pt-BR&sa=X&ei=OgoKVNLCJIGRgwS0i4GwDw&ved=0CB0Q6AEwAA#v=onepage&q=De%20acordo%20com%20Marshall%2C%20existem%20algumas%20áreas-alvo%20para%20a%20avaliação%20de%20agressores%20sexuais.&f=false

  • Gab. C

    "Os ofensores sexuais intrafamiliares e extrafamiliares possuem um perfil diferente. Uma metanálise realizada por Seto et al. (2015) apontou que os ofensores extrafamiliares são mais jovens, desempregados e possuem menores escores de inteligência, mas maior escolaridade em relação aos intrafamiliares. Ainda, apresen infância: eles apresentam taram menor dificuldade na menor probabilidade de ser vítima de abuso sexual, negligência familiar ou vínculo pobre com as mães. Eles possuem mais problemas de regulação emocional, crenças suportivas ao abuso sexual, maior histórico criminal, menor empatia com a vítima, maior hostilidade perante mulheres, são mais propensos a se identificar emocionalmente com crianças ou adolescentes , negar suas ofensas sexuais ou minimizar suas ofensas perpetradas. Os ofensores extrafamiliares tam sexualidade mais atípica (Seto et al., 2015)

    Fonte: chrome-extension://efaidnbmnnnibpcajpcglclefindmkaj/viewer.html?pdfurl=https%3A%2F%2Fprceu.usp.br%2Fwp-content%2Fuploads%2F2021%2F04%2F2020_RaianeNunesNogueira.pdf&clen=1279683&chunk=true


ID
1265896
Banca
TJ-PR
Órgão
TJ-PR
Ano
2013
Provas
Disciplina
Psicologia
Assuntos

Sobre o trabalho final do processo de avaliação, considere os seguintes itens:

1. Relatório/laudo psicológico.
2. Orientação para a intervenção do psicólogo.
3. Parecer psicológico.
4. Declaração psicológica.

O trabalho final do processo de avaliação psicológica no contexto forense pode gerar:

Alternativas

ID
1265902
Banca
TJ-PR
Órgão
TJ-PR
Ano
2013
Provas
Disciplina
Psicologia
Assuntos

Sobre as avaliações na área jurídica e da psicologia, numere a coluna da direita de acordo com sua correspondência com a coluna da esquerda.

1. O foco é a relação decorrente dos achados clínicos com os constructos legais.
2. O objetivo é a compreensão do mundo interno do paciente.
3. Refere-se à não receptividade de tratamentos.
4. Refere-se às condições de compreensão do acusado nos procedimentos e consequências das decisões legais.
5. Refere-se à diferenciação do comportamento delinquente não apenas de uma doença mental potencial, mas também da imaturidade do jovem.

( ) Avaliação forense.
( ) Avaliação clínica.
( ) Avaliação para transferência.
( ) Avaliação de inimputabilidade.
( ) Avaliação de capacidade.

Assinale a alternativa que apresenta a numeração correta da coluna da direita, de cima para baixo.

Alternativas
Comentários
  • Refere-se à não receptividade de tratamentos? Pra mim a atividade clínica está muito mais ligada a essa questão: "O objetivo é a compreensão do mundo interno do paciente". Tantos motivos para se fazer uma avaliação clínica e a questão só traz a não receptividade de tratamentos, não acho que deva ser colocada como a principal.

  • Gostaria de alguma referência a esta questão, caso alguem saiba.

  • embora tenha acertado por dedução, já que tinha certeza que a penúltima era o 5 e a última o 4, achei a questão muito mal formulada e dirigida ao erro.


ID
1265914
Banca
TJ-PR
Órgão
TJ-PR
Ano
2013
Provas
Disciplina
Psicologia
Assuntos

Segundo Rovinski (2011, p. 100), “a regra básica para que seja mantida a ética na elaboração de documentos psicológicos na área forense, é que se respeite o conceito de pertinência”.

Assinale a alternativa que justifica corretamente esse conceito.

Alternativas
Comentários
  • a) Devem-se incluir, sem restrições, todos os dados subjetivos obtidos no processo de avaliação psicológica realizado.

     

    De acordo com a Resolução 007/2003 é preciso observar os cuidados em relação aos deveres do psicólogo nas suas relações com a pessoa atendida, ao sigilo profissional, às relações com a justiça e ao alcance das informações - identificando riscos e compromissos em relação à utilização das informações presentes nos documentos em sua dimensão de relações de poder.

     

    b) Devem-se abordar aspectos que vão além da questão jurídica, para dar significado aos dados obtidos na avaliação psicológica.

     

    De acordo com a Resolução 007/2003 a linguagem nos documentos deve ser precisa, clara, inteligível e concisa, ou seja, deve-se restringir pontualmente às informações que se fizerem necessárias, recusando qualquer tipo de consideração que não tenha relação com a finalidade do documento específico.

    c) Devem-se descrever na sua essência conteúdos extraídos das entrevistas estruturadas, com o cuidado de não interferir no que foi relatado.

     

    De acordo com a Resolução 007/2003 a linguagem nos documentos deve ser precisa, clara, inteligível e concisa, ou seja, deve-se restringir pontualmente às informações que se fizerem necessárias, recusando qualquer tipo de consideração que não tenha relação com a finalidade do documento específico.


ID
1318300
Banca
FGV
Órgão
TJ-GO
Ano
2014
Provas
Disciplina
Psicologia
Assuntos

A Resolução CFP nº 008/2010 foi criada com o objetivo de dispor sobre a atuação do psicólogo como perito e assistente técnico no Poder Judiciário. Segundo tal documento:

Alternativas
Comentários
  • A) ERRADA. Art. 2º - O psicólogo assistente técnico não deve estar presente durante a realização dos procedimentos metodológicos que norteiam o atendimento do psicólogo perito e vice-versa, para que não haja interferência na dinâmica e qualidade do serviço realizado.

    B) ERRADA. Parágrafo Único - A relação entre os profissionais deve se pautar no respeito e colaboração, cada qual exercendo suas competências, podendo o assistente técnico formular quesitos ao psicólogo perito.

    C) ERRADA. Art. 3º - Conforme a especificidade de cada situação, o trabalho pericial poderá contemplar observações, entrevistas, visitas domiciliares e institucionais, aplicação de testes psicológicos, utilização de recursos lúdicos e outros instrumentos, métodos e técnicas reconhecidas pelo Conselho Federal de Psicologia.

    D) CORRETA. Art. 7º - Em seu relatório, o psicólogo perito apresentará indicativos pertinentes à sua investigação que possam diretamente subsidiar o Juiz na solicitação realizada, reconhecendo os limites legais de sua atuação profissional, sem adentrar nas decisões, que são exclusivas às atribuições dos magistrados

    E) ERRADA. Art. 10 - Com intuito de preservar o direito à intimidade e equidade de condições, é vedado ao psicólogo que esteja atuando como psicoterapeuta das partes envolvidas em um litígio:

    I - Atuar como perito ou assistente técnico de pessoas atendidas por ele e/ou de terceiros envolvidos na mesma situação litigiosa;

    II – Produzir documentos advindos do processo psicoterápico com a finalidade de fornecer informações à instância judicial acerca das pessoas atendidas, sem o consentimento formal destas últimas, à exceção de Declarações, conforme a Resolução CFP nº 07/2003.


     

  • RESOLUÇÃO CFP Nº 008/2010

    Art. 2º - O psicólogo assistente técnico não deve estar presente durante a realização dos procedimentos metodológicos que norteiam o atendimento do psicólogo perito e vice-versa, para que não haja interferência na dinâmica e qualidade do serviço realizado (A)

    Art. 2º§ único -  A relação entre os profissionais deve se pautar no respeito e colaboração, cada qual exercendo suas competências, podendo o assistente técnico formular quesitos ao psicólogo perito (B)

    Art. 3º - Conforme a especificidade de cada situação, o trabalho pericial poderá contemplar observações, entrevistas, visitas domiciliares e institucionais, aplicação de testes psicológicos, utilização de recursos lúdicos e outros instrumentos, métodos e técnicas reconhecidas pelo Conselho Federal de Psicologia (C)

    Art. 7º - Em seu relatório, o psicólogo perito apresentará indicativos pertinentes à sua investigação que possam diretamente subsidiar o Juiz na solicitação realizada, reconhecendo os limites legais de sua atuação profissional, sem adentrar nas decisões, que são exclusivas às atribuições dos magistrados (D)

    Art. 10 inciso I - Com intuito de preservar o direito à intimidade e equidade de condições, é vedado ao psicólogo que esteja atuando como psicoterapeuta das partes envolvidas em um litígio atuar como perito ou assistente técnico de pessoas atendidas por ele e/ou de terceiros envolvidos na mesma situação litigiosa (E)

    Gabarito: D


ID
1318336
Banca
FGV
Órgão
TJ-GO
Ano
2014
Provas
Disciplina
Psicologia
Assuntos

O Sr. João Silva, 66 anos, foi denunciado por fazer uma ligação clandestina de água no condomínio em que é síndico. Considerando a legislação no que concerne às pessoas com idade igual ou superior a 60 (sessenta) anos, o Sr. João Silva será:

Alternativas
Comentários
  • LETRA D,  d) capaz de responder por seus atos;


    porém esta questão não é de Psicologia e sim de Direito

  • Estatuto do idoso.

  • Bruna, essa questão é de Psicologia Jurídica

ID
1324633
Banca
CESPE / CEBRASPE
Órgão
INCA
Ano
2010
Provas
Disciplina
Psicologia
Assuntos

     Uma criança, cinco anos de idade, internada em hospital para tratamento de leucemia mieloblástica aguda, apresenta quadro de anemia intensa. A equipe médica prescreve transfusão sanguínea, mas os pais recusam tal procedimento.

Com base nesse caso clínico, julgue o item que se segue.

Mesmo existindo conflitos de valores ou de princípios paternos com a equipe de saúde, o pátrio poder não poderá ser confrontado ética e legalmente nos tribunais.

Alternativas
Comentários
  • Tratando-se de menor tudo pode...menos fazer algo errado


ID
1369360
Banca
FGV
Órgão
TJ-RJ
Ano
2014
Provas
Disciplina
Psicologia
Assuntos

Texto disponibilizado no Portal Ipea mostra que o Brasil vive uma fase de envelhecimento. De acordo com o artigo, entre os anos de 1940 e 2010, a população com mais de 60 anos aumentou de 4% para 11%. A expectativa é que o grupo formado por 20,6 milhões de idosos em 2010 chegue a 57 milhões até 2040, ou seja, 28% da população. O destino desses idosos tem sido cada vez mais decidido pela Justiça brasileira através do aumento dos pedidos de interdição. Constitui motivo legítimo para o deferimento do pedido de interdição dessa população:

Alternativas
Comentários
  • Da Ação de Interdição

    Trata-se de uma ação intentada no âmbito cível e tem por fim a declaração da incapacidade de determinada pessoa. É a ação na qual se requer  seja declarada a incapacidade de uma pessoa comandar seus atos na vida civil e, consequentemente, seja nomeado um curador para a mesma. O Código Cívil , no art. 1.767 enumera aqueles que estão sujeitos à curatela, ou seja, incapazes aptos à interdição:

    a) os psicopatas

    b) os surdos-mudos sem educação que os habilite a enunciar precisamente sua vontade

    c)os pródigos( consumidores compulsivos)

    d) toxicômanos (usuário frequente de drogas) acometidos de pertubações mentais.

    El-Jaick(s.d). Da Ação de Interdição.Série Aperfeiçoamento de Magistrados. Curso: Processo Civil- Procedimentos Especiais.

  • GABARITO: a demência neurodegenerativa;

  • A questão  está referenciada em uma versão desatualizada do Código Civil que foi alterada após a publicação do Estatuto da Pessoa com Deficiência. Vejamos:

    Art. 1.767. Estão sujeitos a curatela:

    I - aqueles que, por enfermidade ou deficiência mental, não tiverem o necessário discernimento para os atos da vida civil;

    II - aqueles que, por outra causa duradoura, não puderem exprimir a sua vontade;

    III - os deficientes mentais, os ébrios habituais e os viciados em tóxicos;

    IV - os excepcionais sem completo desenvolvimento mental;

    I - aqueles que, por causa transitória ou permanente, não puderem exprimir sua vontade;    (Redação dada pela Lei nº 13.146, de 2015) (Vigência)

    II - (Revogado) ;    (Redação dada pela Lei nº 13.146, de 2015)   (Vigência)

    III - os ébrios habituais e os viciados em tóxico;      (Redação dada pela Lei nº 13.146, de 2015)   (Vigência)

    IV - (Revogado) ;     (Redação dada pela Lei nº 13.146, de 2015)   (Vigência)

    V - os pródigos.


    Por força dessas alterações não existem mais absolutamente incapazes maiores. Sendo assim, a curatela somente incide para os maiores relativamente incapazes, que, na nova redação do artigo são os ébrios habituais (no sentido de alcoolismo); viciados em tóxicos; as pessoas que, por causa transitória ou definitiva, não puderem exprimir vontade; e os pródigos (no sentido de dissipadores de bens).

    GABARITO PROFESSOR: SEM RESPOSTA, QUESTÃO DESATUALIZADA.
    GABARITO BANCA: B

  • a demência neurodegenerativa;


ID
1403287
Banca
FGV
Órgão
TJ-BA
Ano
2015
Provas
Disciplina
Psicologia
Assuntos

“Vivemos numa época em que um notório desenvolvimento científico mescla-se a um silenciamento ou a uma espetacularização acerca da morte. (...) Em tal contexto o velho passa a ser um sujeito vazio, impossível de ser compreendido, em relação ao qual não se tem sequer paciência. (...) O idoso é jogado para as margens da experiência social e cultural, e seu acolhimento dependerá de sua incorporação ao mercado.” (AGRA DO Ó, A. Norbert Elias e uma narrativa acerca do envelhecimento e da morte. Apud COIMBRA, J.C. O psicólogo, o sistema de justiça e os casos de violência intrafamiliar contra idosos. In BRANDÃO, E. et GONÇALVES, H. S. Psicologia Jurídica no Brasil. Rio de Janeiro: NAU, 2011).

Verificamos que apesar da promulgação da Lei nº 10.741/2003 (Estatuto do Idoso), e da criação de Varas do Idoso em alguns estados brasileiros, tais Varas, majoritariamente, não contam com equipes técnicas lotadas nestas serventias. Tal fenômeno pode ser compreendido pela:

Alternativas
Comentários
  • ESTATUTO DO IDOSO 

    Art. 70.O Poder Público poderá criar varas especializadas e exclusivas do idoso.

    Quando você promulga que poderá, deixa margem para que algumas questões fiquem relativamente soltas. A única previsão legal que existe em atenção ao idoso é relacionada a saúde no tocante ao atendimento domiciliar.

    http://www.sdh.gov.br/assuntos/pessoa-idosa/publicacoes/estatuto-do-idoso-dignidade-humana-como-foco

  • Tania, para complementar... O Art.15, SS5 que cita a Lei 12.896 de 2013...

  • Gab D

    falta de previsão legal para atuação da equipe interprofissional nestas Varas;


ID
1403293
Banca
FGV
Órgão
TJ-BA
Ano
2015
Provas
Disciplina
Psicologia
Assuntos

O Sr. José Alfredo (66 anos) foi encontrado em um barraco pela Vigilância Sanitária desamparado e em precárias condições de saúde. Diante dos riscos a que vinha sendo exposto, foi colocado em instituição de acolhimento que atende idosos. Estudos psicossociais apontaram que o Sr. José Alfredo possui dois filhos, Antônio (30 anos) e Manoel (32 anos), e esposa, Sra. Ana Maria (55 anos), de quem está separado de corpos, mas não divorciado ou separado judicialmente. Em razão dessa informação, cogitou- se a propositura de ação de alimentos face aos filhos e à esposa, mas o idoso recusa peremptoriamente que a ação seja movida perante seu filho Manoel e sua esposa, desejando que apenas Antônio figure como polo passivo da ação. Segundo o idoso, Antônio seria um empresário bem sucedido, com boas condições financeiras, enquanto Manoel seria profissional autônomo da construção civil, que padece com dificuldades para sobreviver, o mesmo ocorrendo com sua esposa. À luz do Estatuto do Idoso, o Sr. José Alfredo:

Alternativas
Comentários
  • e)

    pode ser atendido em seu desejo, pois a obrigação alimentar é solidária, podendo o idoso optar entre os prestadores.

  • CAPÍTULO III

    Dos Alimentos

           Art. 11. Os alimentos serão prestados ao idoso na forma da lei civil.

           Art. 12. A obrigação alimentar é solidária, podendo o idoso optar entre os prestadores

          Art. 13. As transações relativas a alimentos poderão ser celebradas perante o Promotor de Justiça ou Defensor Público, que as referendará, e passarão a ter efeito de título executivo extrajudicial nos termos da lei processual civil.        

           Art. 14. Se o idoso ou seus familiares não possuírem condições econômicas de prover o seu sustento, impõe-se ao Poder Público esse provimento, no âmbito da assistência social.

  • LEI Nº 10.741/2003

     

     Art. 12 –  A obrigação alimentar é solidária, podendo o idoso optar entre os prestadores.

     

    Quem escolheu a busca não pode recusar a travessia - Guimarães Rosa

    ------------------- 

    Gabarito: E

  • GABARITO LETRA E


ID
1403302
Banca
FGV
Órgão
TJ-BA
Ano
2015
Provas
Disciplina
Psicologia
Assuntos

Tereza, 83 anos, viúva, pensionista militar, reside sozinha em seu apartamento e ocupa seu dia com atividades filantrópicas, religiosas e culturais. Seu filho Álvaro compareceu à Vara especializada em idosos requerendo orientação sobre os procedimentos para torná-lo curador da idosa. Considerando na análise desse caso o disposto na legislação que trata do direito do idoso:

Alternativas

ID
1405255
Banca
FUNCAB
Órgão
SEDS-TO
Ano
2014
Provas
Disciplina
Psicologia
Assuntos

As visitas domiciliares são estratégias importantes para a abordagem de famílias em situação de vulnerabilidade, pois consistem em uma excelente oportunidade para:

Alternativas

ID
1409029
Banca
FUNCAB
Órgão
SEDS-TO
Ano
2014
Provas
Disciplina
Psicologia
Assuntos

Na abordagem com famílias em situação de vulnerabilidade, os diferentes pontos de vista sobre a situação de violência devem ser valorizados, refletindo-se:

Alternativas

ID
1409035
Banca
FUNCAB
Órgão
SEDS-TO
Ano
2014
Provas
Disciplina
Psicologia
Assuntos

Pode-se considerar que a conduta desviante não tem seu foco apenas no sujeito e na organização dos seus traços de personalidade, mas é salientada e influenciada a partir da administração e socialização, ou seja:

Alternativas
Comentários
  • Segundo Formiga et al (2008),

     

    "Pode-se inferir que a conduta desviante, seja ela anti-social, seja delitiva, não teria seu foco apenas no sujeito e na organização dos traços de personalidade, mas que essas condutas seriam salientadas e influenciadas a partir da administração e socialização que os traços da busca de sensação estivessem envolvidos nas relações de afiliação endogrupal, isto é, a capacidade com o sujeito se organiza a partir da identidade com os pares (colegas, vizinhos, etc.) ou com os responsáveis pela manutenção de uma conduta convencional (pais, professores, etc.)."

     

    Quem escolheu a busca não pode recusar a travessia - Guimarães Rosa

    ------------------- 

    Gabarito: A


ID
1409038
Banca
FUNCAB
Órgão
SEDS-TO
Ano
2014
Provas
Disciplina
Psicologia
Assuntos

Uma das funções privativas do psicólogo é a solução de problemas de ajustamento, sobre o qual entende-se que é o processo:

Alternativas
Comentários
  • TÍTULO I

    DA CARACTERIZAÇÃO DA PROFISSÃO

    Art. 2º - Os métodos e as técnicas psicológicas utilizados no exercício das funções privativas do Psicólogo a que se refere o § 1º do art. 12 da Lei nº 4.119, de 27 de agosto de 1962, são entendidos da seguinte forma:

    VIII - SOLUÇÃO DE PROBLEMAS DE AJUSTAMENTO - é o processo que propicia condições de auto-realização, de convivência e de desempenho para o indivíduo, o grupo, a instituição e a comunidade, mediante métodos psicológicos preventivos, psicoterápicos e de reabilitação.

     

    http://www.crpsp.org.br/portal/orientacao/resolucoes_cfp/fr_cfp_018-00.aspx


ID
1409065
Banca
FUNCAB
Órgão
SEDS-TO
Ano
2014
Provas
Disciplina
Psicologia
Assuntos

Atualmente debate-se no Brasil a redução da maioridade penal.Um dos argumentos desfavoráveis a essamudança é:

Alternativas
Comentários
  • https://18razoes.wordpress.com/quem-somos/

     

    Esse documento é referência para quem deseja saber mais acerca do assunto

     

    Quem escolheu a busca não pode recusar a travessia - Guimarães Rosa

    ------------------- 

    Gabarito: C


ID
1409092
Banca
FUNCAB
Órgão
SEDS-TO
Ano
2014
Provas
Disciplina
Psicologia
Assuntos

O uso intencional da força física ou do poder, real ou emameaça, contra si próprio, contra outra pessoa, ou contra um grupo ou uma comunidade, que resulte ou tenha grande possibilidade de resultar em lesão, morte, dano psicológico, deficiência de desenvolvimento ou privação pode ser definido como:

Alternativas

ID
1455358
Banca
FGV
Órgão
TJ-SC
Ano
2015
Provas
Disciplina
Psicologia
Assuntos

“Os conhecimentos que construímos estão embebidos no contexto temporal, cultural, espacial em que são criados e, assim, considera-se que as formações da subjetividade não podem ser compreendidas desligadas da formação social na qual se constituem." (MANCEBO, D. Indivíduo e Psicologia. In JACÓ- VILELA, A. (org.) Psicologia Social: abordagens sócio-históricas e desafios contemporâneos. Rio de Janeiro: Eduerj, 1999).

Na perspectiva histórica, a Psicologia Jurídica constituiu-se como:

Alternativas
Comentários
  • O ponto chave da questão se dá justamente na solicitação feita. Não está diretamente relacionado ao texto e ao livro. Mas é necessário lembrar a atuação do perito no âmbito jurídico e o psicodiagnóstico.

    "considera-se que as formações da subjetividade não podem ser compreendidas desligadas da formação social na qual se constituem"

  • Gabarito: A

    "Ao analisar os campos de atuação do psicólogo jurídico, percebe-se um predomínio da atuação desses profissionais enquanto avaliadores. A elaboração de psicodiagnósticos, presente desde o surgimento da Psicologia Jurídica, permanece como um forte campo de exercício profissional".

    LAGO, V. et al. Um breve histórico da psicologia jurídica no Brasil e seus campos de atuação. Estudos de Psicologia I Campinas I 26(4) I 483-491 I outubro - dezembro 2009.

  • Foi-se o tempo que a prática da psicologia jurídica era um saber centrado principalmente no psicodiagnóstico. Está precisando se atualizar banca FGV. 

  • Assim, tá meio desatualizada a questão e restrita, mas se acerta por eliminação..

     

  • Galera, também errei a questão, mas por falta de atenção. Observem a pergunta:

     

    Na perspectiva histórica, a Psicologia Jurídica constituiu-se como:

     

    Ou seja a perspectiva histórica remete ao passado da psicologia jurídica!

     

    Bons estudos!

  • Gabarito A

    A questão pede a resposta em relação ao contexto histórico, e a Psicologia Jurídica constituiu-se como:

    um saber centrado principalmente na práxis do psicodiagnóstico


ID
1455361
Banca
FGV
Órgão
TJ-SC
Ano
2015
Provas
Disciplina
Psicologia
Assuntos

“As primeiras referências à utilização do discurso “psi" na sociedade brasileira datam das primeiras décadas do século XX, pouco após a criação do Juízo de Menores, em 1923, e da promulgação do Código de menores de 1927. (...) É digno de nota que (...) estivesse representada a nata da sociedade brasileira (...), por seu intermédio a sociedade foi apresentada às teorias mais avançadas da época." (SANTOS, E. P. S. Desconstruindo a menoridade: a psicologia e a produção da categoria menor. In BRANDÃO, E. et GONÇALVES, H. S. Psicologia Jurídica no Brasil. Rio de Janeiro: NAU, 2011).

Naquele contexto social, a psicanálise foi valorizada como um saber:


Alternativas
Comentários
  • "Entre outros saberes, “a psicanálise (era)”, nas palavras de Nunes (1992: 72), "valorizada enquanto  um saber que poderia se tornar instrumento útil para os programas de eugenia (...), O que interessava era a possibilidade que alguns de seus postulados abririam para o projeto de controle e transformação dos indivíduos" In BRANDÃO, E. et GONÇALVES, H. S.Psicologia Jurídica no Brasil. Rio de Janeiro: NAU, 2011).

    Questãozinha ein.....

    letra B

  • questão horrível!!

  • questãozinha bem ruim, ein...

  • Sobre a fundação da Eugenia, a título de informação:

    "O objetivo último de Galton era encorajar o nascimento de indivíduos mais eminentes ou capazes e desencorajar o nascimento dos incapazes. Para ajudar a atingir essa meta, ele fundou a ciência da eugenia (a ciência que trata dos fatores capazes de aprimorar as qualidades hereditárias da raça humana), afirmando que os seres humanos, assim como os animais, podiam ser aperfeiçoados por seleção artificial. Ele acreditava que, se homens e mulheres de talento considerável fossem selecionados e acasalados por sucessivas gerações, seria produzida uma raça de pessoas altamente dotadas. Propunha Galton que se desenvolvessem testes de inteligência a ser usados na escolha dos homens e mulheres mais brilhantes para o acasalamen to seletivo, recomendando que quem alcançasse os níveis mais altos nos testes devia receber incentivos financeiros para se casar e ter filhos. (É interessante o fato de Galton, que fundou a eugenia e acreditava que só as pessoas muito inteligentes deviam se reproduzir, não ter tido filhos. Ao que parece, o problema era genético; nenhum dos seus irmãos os teve.)"

    Fonte: História da Psicologia Moderna - Shultz


ID
1455394
Banca
FGV
Órgão
TJ-SC
Ano
2015
Provas
Disciplina
Psicologia
Assuntos

Sidney Shine no livro Avaliação Psicológica e Lei escreve que “A avaliação psicológica em contexto forense ou jurídico precisa ser reconhecida pelo que ela é: uma modalidade específica de avaliação com características intrínsecas ao seu objeto e objetivo”. Em comparação com a avaliação psicológica no contexto clínico, a especificidade da avaliação psicológica no contexto jurídico é determinada pelo (a):

Alternativas
Comentários
  • O juiz.

  • Origem da demanda: na clínica, geralmente é a pessoa ou familiar que procura a avaliação psicológica e, na forense é, usualmente, o advogado ou o Tribunal.

    Em comparação com a avaliação psicológica no contexto clínico, a especificidade da avaliação psicológica no contexto jurídico é determinada pelo (a):

  • c) implicação de um terceiro na relação de trabalho entre o psicólogo e o sujeito;


  • Alternativa C. Quando a resposta menciona um terceiro na relação é a Justiça, a figura do Juiz.

  • O uso dos testes psicológicos, entrevistas e o registro documental são características da avaliação psicológica, independentemente do contexto, apenas a ESCOLHA de QUAL material utilizado no processo depende do contexto.


    A busca voluntária pela avaliação psicodiagnóstica pode ocorrer na área clínica, nunca na jurídica, e o sujeito é o mais interessado, restando assim, a alternativa C, onde o terceiro é o sistema judiciário.

  • A terceira pessoa implicada no processo é o JUIZ.

    ITEM C


ID
1480129
Banca
FCC
Órgão
MPE-MA
Ano
2013
Provas
Disciplina
Psicologia
Assuntos

O Código Civil Brasileiro distingue a incapacidade civil entre absoluta e relativa. Esse conceito apresenta estreita relação com a saúde mental e atribui àqueles considerados incapazes a

Alternativas

ID
1480132
Banca
FCC
Órgão
MPE-MA
Ano
2013
Provas
Disciplina
Psicologia
Assuntos

A Psicologia Jurídica ou Judiciária, tal qual se conhece hoje, é fruto de uma aproximação entre Psicologia e Direito, que ocorreu especificamente ao final do século XIX com a denominada

Alternativas
Comentários
  • A psicologia do testemunho tem origem nos trabalhos de Alfred Binet, ela é uma subárea da psicologia jurídica, e começou estudando a sugestionabilidade da memória das crianças. 

     

    http://mundodapsi.com/psicologia-juridica/ 


ID
1480135
Banca
FCC
Órgão
MPE-MA
Ano
2013
Provas
Disciplina
Psicologia
Assuntos

A ação aplicada quando uma determinada pessoa, em virtude de suas condições psíquicas alteradas, perde a capacidade de gerir seus bens e sua própria pessoa é denominada de

Alternativas
Comentários
  • Encontrei uma fácil da FCC  kkk

  • alimentos? tá de sacanagem rs

ID
1480138
Banca
FCC
Órgão
MPE-MA
Ano
2013
Provas
Disciplina
Psicologia
Assuntos

Assinale a alternativa INCORRETA.

Alternativas

ID
1691077
Banca
FEPESE
Órgão
MPE-SC
Ano
2014
Provas
Disciplina
Psicologia
Assuntos

A respeito das falsas memórias e seu processo de implantação no(s) filho(s), analise as afirmativas a seguir:

1. O filho é convencido da existência de determinados fatos e levado a repetir o que lhe é afirmado como tendo realmente acontecido.

2. O filho consegue discernir que está sendo manipulado, porém passa a acreditar naquilo que lhe é dito de forma insistente e repetida, por temer represálias por parte do genitor manipulador.

3. O processo de implantação de falsas memórias produz um quadro tal que, com o tempo, nem o genitor manipulador distingue mais a diferença entre verdade e mentira, e a sua verdade passa a ser verdade para o filho.

4. A implantação de falsas memórias produz como resultado que o filho manipulado passe a viver com falsas personagens de uma falsa existência, pois foi convencido, por um processo de “lavagem cerebral", de que aquilo que lhe foi dito e repetido de fato ocorreu.

5. As falsas memórias, embora decorram da concordância do filho com as “verdades" formuladas e repetidas a ele pelo genitor manipulador a respeito do outro genitor, incluem apenas a alusão a situações fantasiosas e não a fatos realmente acontecidos.

Assinale a alternativa que indica todas as afirmativas corretas.


Alternativas

ID
1775287
Banca
FUNIVERSA
Órgão
Secretaria da Criança - DF
Ano
2015
Provas
Disciplina
Psicologia
Assuntos

A possibilidade de resolução de conflitos não se resume à esfera judiciária. A mediação é um dos mecanismos adotados para a solução de divergências fora do âmbito judicial. No que se refere à mediação de conflitos, assinale a alternativa correta.

Alternativas
Comentários
  • O mediador NÃO possui poder de decisão. Ele auxilia para a restauração do dialógo entre as partes a fim de levar a uma decisão consensual das mesmas.

  • A) O mediador é um facilitador que possui o poder de decisão e que atua no processo de mediação com o intuito de auxiliar as partes na busca de uma solução comum. (Não possui poder de decisão)

    B) Mediação é o mesmo que conciliação. (conciliação: ato ou efeito de apaziguar-se com; pacificação, acomodação, reconciliação./ mediação: ato de servir de intermediário entre pessoas ou grupos; intervenção, intermédio.)

    C) Um dos benefícios da mediação consiste na busca de soluções que favoreçam ambas as partes, o que caracteriza uma díade do tipo “ganha-ganha” e rompe com o paradigma de que todo conflito deve gerar vencedores e perdedores.

    D) A flexibilidade, a passividade, a disposição para a escuta, a paciência, a serenidade e a parcialidade são características desejáveis ao mediador. (imparcial)

    E) Caso ocorra a resistência da participação de uma das partes na mediação, faz-se necessário que o mediador utilize estratégias de convencimento do indivíduo no sentido de demovê-lo dessa resistência.

  • Vejamos a correção das assertivas incorretas:

    • a) na mediação, o poder de decisão é das partes e não do mediador;
    • b) mediação e conciliação são distintos métodos autocompositivos de resolução de conflitos;
    • d) uma das principais características da mediação é a presença de um terceiro imparcial;
    • e) um dos princípios da mediação é a voluntariedade;

    Gabarito: C


ID
1797754
Banca
CESPE / CEBRASPE
Órgão
TJ-DFT
Ano
2015
Provas
Disciplina
Psicologia
Assuntos

No contexto forense, o uso de técnicas de entrevistas e a aplicação de treinamentos para entrevistadores têm possibilitado uma melhoria no desempenho dos psicólogos durante a realização de entrevistas, especificamente na detecção de mentiras. Considerando esse assunto, julgue o item que se segue com relação à entrevista forense.

O método PEACE, desenvolvido pela Associação dos Chefes de Polícia da Inglaterra e País de Gales e pelo Home Office, é utilizado para entrevistar suspeitos e testemunhas, com base nos conhecimentos da área de psicologia e em evidências obtidas por meio das pesquisas psicológicas, que possibilita treinar os profissionais para que identifiquem sinais comportamentais de mentiras.

Alternativas
Comentários
  • Uma saída possível ao dilema de identificação de mentiras seria utilizar mais a “análise do discurso” do entrevistado (VRIJ, 2008; VRIJ et al., 2004). Ou seja, nessa perspectiva, o entrevistador deveria “escutar mais do que olhar”, criando um espaço de fala para que o entrevistado vá dando o seu relato, o mais detalhado possível. 

    Seguindo nesse caminho, a Associação de Chefes de Polícia da Inglaterra e País de Gales e o Home Office desenvolveram o método “PEACE”, colocado em atividade nos anos 1990, o qual enfatiza o treinamento em técnicas de entrevista investigativa. (BALDWIN, 1992). 

    Os pressupostos do método Problema, Emoção, Análise, Contemplação e Equilíbrio (PEACE) (livremente adaptados ao português) são: P = planejar antecipadamente a entrevista. 
    E = engajar o entrevistado na conversação. 
    A = acessar o relato livre (sem interrupção e coerção). 
    C = cerrar (fechar) a entrevista realizando um resumo. 
    E = expandir os dados colhidos (avaliar o material após a entrevista). 

  • Não concordo com o gabarito 

    o método vai potencializar a fala do investigado e a partir dele fazer essa análise do discurso. "Uma saída possível ao dilema de identificação de mentiras seria utilizar mais a “análise do discurso” do entrevistado (VRIJ, 2008; VRIJ et al., 2004). Ou seja, nessa perspectiva, o entrevistador deveria “escutar mais do que olhar”, criando um espaço de fala para que o entrevistado vá dando o seu relato, o mais detalhado possível.

    Além disso, no livro da Rovinski ela aponta que "após a implementação do PEACE, a associação dos policiais britânicos decidiu não mais treinar policiais nos sinais comportamentais de mentira"

  • ??????????????

  • Discordo do gabarito, com base no seguinte trecho do livro Psicologia Jurídica: Perspectivas Teóricas e Processos de Intervenção:

    "Além de implementar a capacitação de policias britânicos no PEACE, baseado em evidências de pesquisas psicológicas, a Associação dos Plociais Britânicos decidiu não mais treinar os policias nos sinais comportamentais de mentira."

    (Sonia Liane Reichert Rovinski,Roberto Moraes Cruz)

     

  • Uma saída possível ao dilema de identificação de mentiras seria utilizar mais a “análise do discurso” do entrevistado (VRIJ, 2008; VRIJ et al., 2004). Ou seja, nessa perspectiva, o entrevistador deveria “escutar mais do que olhar”, criando um espaço de fala para que o entrevistado vá dando o seu relato, o mais detalhado possível. 

    Seguindo nesse caminho, a Associação de Chefes de Polícia da Inglaterra e País de Gales e o Home Office desenvolveram o método “PEACE”, colocado em atividade nos anos 1990, o qual enfatiza o treinamento em técnicas de entrevista investigativa. (BALDWIN, 1992). 

    Os pressupostos do método Problema, Emoção, Análise, Contemplação e Equilíbrio (PEACE) (livremente adaptados ao português) são: 
    P = planejar antecipadamente a entrevista. 
    E = engajar o entrevistado na conversação. 
    A = acessar o relato livre (sem interrupção e coerção). 
    C = cerrar (fechar) a entrevista realizando um resumo. 
    E = expandir os dados colhidos (avaliar o material após a entrevista). 

    Pesquisas com esse método (BULL; CHERRYMAN, 1996; MEMON; BULL, 1999) apontaram que o PEACE é mais eficaz quando o entrevistador consegue manifestar as seguintes habilidades: 
    1. Escuta (com emprego apropriado de silêncio). 
    2. Preparação antes da entrevista. 
    3. Questionar de forma adequada (com uso de perguntas abertas). 
    4. Conhecimento do caso. 
    5. Flexibilidade. 
    6. Capacidade de se relacionar com o entrevistado. 
    7. Empatia. 
    8. Manter o entrevistado em tópicos importantes. 
    9. Responder o que o entrevistado pergunta. 
    10. Não revelar todas as informações do caso logo no início da entrevista. 

    Outra pesquisa para gerar novas avaliações do PEACE foi realizada na Inglaterra (SOUKARA; BULL, VRIJ, 2002) e os achados indicaram que: 
    1. Tanto o planejamento e preparação para a entrevista, quanto a habilidade do entrevistador são de extrema importância. 
    2. A categoria do crime e os atributos do suspeito devem influenciar o modo de escolha da estratégia de entrevista. 
    3. As evidências devem determinar o tipo de entrevista. 
    4. A entrevista deve ter como objetivo principal buscar os fatos e não uma mera confissão. 


ID
1797757
Banca
CESPE / CEBRASPE
Órgão
TJ-DFT
Ano
2015
Provas
Disciplina
Psicologia
Assuntos

No contexto forense, o uso de técnicas de entrevistas e a aplicação de treinamentos para entrevistadores têm possibilitado uma melhoria no desempenho dos psicólogos durante a realização de entrevistas, especificamente na detecção de mentiras. Considerando esse assunto, julgue o item que se segue com relação à entrevista forense.

Os pressupostos da abordagem PEACE para entrevistas investigativas são os seguintes: P, de planejamento e preparação; E, de engajamento do suspeito; A, de aquisição ou obtenção de relato livre do suspeito, sem interrupções; C, de cerrar, ou seja, de fechamento, incluindo o resumo; e E, de expansão dos dados coletados ao se avaliá-los após a entrevista.

Alternativas
Comentários
  • Os pressupostos do método Problema, Emoção, Análise, Contemplação e Equilíbrio (PEACE) (livremente adaptados ao português) são: 
    P = planejar antecipadamente a entrevista. 
    E = engajar o entrevistado na conversação. 
    A = acessar o relato livre (sem interrupção e coerção). 
    C = cerrar (fechar) a entrevista realizando um resumo. 
    E = expandir os dados colhidos (avaliar o material após a entrevista). 


    Fonte: http://www.portaleducacao.com.br/psicologia/artigos/35377/deteccao-de-mentiras-em-psicologia-forense-analise-do-discurso#ixzz44rr4APC1

  • À luz dos achados desses estudos, a Associação dos Chefes de Polícia da
    Inglaterra e País de Gales e o Home Office desenvolveram um novo método
    para a entrevista de suspeitos e testemunhas, que foi chamado de método
    PEACE, que começou a ser implementado no início dos anos 90 (BALDWIN,
    1992). Esta mudança iniciou com o curso de treinamento, de uma semana no
    método PEACE, de todos os policiais responsáveis pelas entrevistas com
    suspeitos. Cabe ressaltar que grande parte deste treinamento estava assentada
    em achados de pesquisas da Psicologia nesta área. Os pressupostos básicos
    dessa nova abordagem PEACE para entrevistas investigativas com suspeito são
    os seguintes: P = planejamento/preparação (antes da entrevista) E = engajar
    suspeito (em conversação) A = obter um relato livre do suspeito (sem
    interrupções) C = fechamento (incluindo o resumo) E = avaliar a entrevista.

    Em:  Rovinsky. Persp teó e proc de intervenção.


ID
1797760
Banca
CESPE / CEBRASPE
Órgão
TJ-DFT
Ano
2015
Provas
Disciplina
Psicologia
Assuntos

No contexto forense, o uso de técnicas de entrevistas e a aplicação de treinamentos para entrevistadores têm possibilitado uma melhoria no desempenho dos psicólogos durante a realização de entrevistas, especificamente na detecção de mentiras. Considerando esse assunto, julgue o item que se segue com relação à entrevista forense.

O método PEACE revela um significativo avanço nos procedimentos de entrevista forense, pois seus pressupostos baseiam-se em valores humanitários e respeito incondicional à vítima entrevistada. Por consequência, o desempenho do psicólogo nessa prática está diretamente relacionado ao quanto esse profissional consegue sustentar o vínculo com o entrevistado nesse momento crucial.

Alternativas
Comentários
  • Este método usa como pressuposto a Análise do discurso do conteúdo relatado pelo entrevistado.

  • O método PEACE foi desenvolvido para entrevistar suspeitos e testemunhas e não vítimas. 

  • Os pressupostos do método Problema, Emoção, Análise, Contemplação e Equilíbrio (PEACE) (livremente adaptados ao português) são: 
    P = planejar antecipadamente a entrevista. 
    E = engajar o entrevistado na conversação. 
    A = acessar o relato livre (sem interrupção e coerção). 
    C = cerrar (fechar) a entrevista realizando um resumo. 
    E = expandir os dados colhidos (avaliar o material após a entrevista). 

    Pesquisas com esse método (BULL; CHERRYMAN, 1996; MEMON; BULL, 1999) apontaram que o PEACE é mais eficaz quando o entrevistador consegue manifestar as seguintes habilidades: 
    1. Escuta (com emprego apropriado de silêncio). 
    2. Preparação antes da entrevista. 
    3. Questionar de forma adequada (com uso de perguntas abertas). 
    4. Conhecimento do caso. 
    5. Flexibilidade. 
    6. Capacidade de se relacionar com o entrevistado. 
    7. Empatia. 
    8. Manter o entrevistado em tópicos importantes. 
    9. Responder o que o entrevistado pergunta. 
    10. Não revelar todas as informações do caso logo no início da entrevista. 
    O método é composto de três elementos (STELLER; KOEHNKEN, 1989): 
    1.
     Entrevista estruturada (como uma entrevista cognitiva). 
    2. Análise de Conteúdo Baseada em Critérios (Criteria Based Content Analysis - CBCA). 
    3. Check-list de validade para analisar o resultado do CBCA. 

    Fonte: https://www.portaleducacao.com.br/conteudo/artigos/educacao/deteccao-de-mentiras-em-psicologia-forense-analise-do-discurso/35377

  • o método PEACE foi desenvolvido a partir de técnicas cognitivas de entrevista, visando detectar mentiras em depoimentos de suspeitos e testemunhas. Esse método é utilizado para a entrevista de suspeitos e testemunhas, e não de vítimas como disse a Banca. Ademais, o objetivo não é criar uma atmosfera humanitária, mas sim extrair elementos que possam identificar se o suspeito ou a testemunha estão falando a verdade ou não.

    FONTE: https://free-content.direcaoconcursos.com.br/demo/curso-8149.pdf


ID
1797763
Banca
CESPE / CEBRASPE
Órgão
TJ-DFT
Ano
2015
Provas
Disciplina
Psicologia
Assuntos

Com relação à atuação do profissional de psicologia nas questões relacionadas ao direito de família, da criança e do adolescente, julgue o item seguinte.

No contexto do divórcio e da dinâmica conjugal e familiar, psicólogos devem estar atentos à estratégia de descrédito, também denominada ziskinização, que diz respeito à tentativa dos envolvidos em desvalorizar as intervenções psicológicas, mesmo diante de evidência científica, por meio de jogos conceituais que misturam hipóteses e recursos de retórica ou, ainda, partem de premissas não generalizáveis e nem sempre verdadeiras.

Alternativas
Comentários
  • Conceito consta no livro de Jorge Trindade, Psicologia jurídica para operadores do direito, capítulo 5.

  • ..... Assim como uma das partes naturalmente buscará aliança com aqueles pontos que parecem favoráveis, a outra, que sentir prejuízo , lutará para obter a sua anulação, colocando em descrédito toda a intervenção pericial desfavorável . Esse mecanismo, conforme relembram Rot, Cartujo, Perez e Atienza ( 1992),'" é conhecido como "estratégia de descrédito" e recebe o nome técnico de ziskinização, expressão que teve origem em Ziskin (1970-80), por sua contribuição a favor da desmitificação do trabalho psicológico.
    De acordo com referidos autores (1992), essa estratégia consiste em desvalorizar as intervenções psicológicas, negando-as mesmo perante a evidência científica, através de puro jogo conceitual que mistura hipóteses e recursos próprios da retórica, partindo de premissas não generalizáveis e nem sempre verdadeiras, escotomizadas e futuristas, dotadas de figuras de linguagem, para retificar ou ratificar uma situação sem importância real, mas capaz de causar dúvida e desarmonia. 

     

    Fonte:  Psicologia jurídica para operadores do direito.Jorge Trindade


ID
1797766
Banca
CESPE / CEBRASPE
Órgão
TJ-DFT
Ano
2015
Provas
Disciplina
Psicologia
Assuntos

Com relação à atuação do profissional de psicologia nas questões relacionadas ao direito de família, da criança e do adolescente, julgue o item seguinte.

Ao intervir em famílias, os psicólogos forenses devem ter a capacidade de reconhecer as fases do ciclo de vida familiar, uma vez que as interseções e tramas de cada família estão presentes no modo como elas enfrentam e superam cada fase, o que tornam visíveis as dificuldades vivenciadas por elas

Alternativas
Comentários
  • A separação pode ser vista como uma das fases do ciclo familiar.Talvez,essa seja uma temática bastante comum em litígios, relacionada a ciclos de desenvolvimento familiar e varas de família:

    As situações envolvendo dissoluções e rompimento do vínculo conjugal é um exemplo dessas condições, que exigem do grupo familiar elaborações e novos ajustamentos, podendo ocorrer de maneira mais desestruturante e dramática para alguns.

    Campbell (1988) sugerem que os fatores determinantes dos impasses familiares que levam ao litígio são: a qualidade do relacionamento do casal na fase de pré-separação, fatores socioambientais que incentivam o litígio e a competição, histórias individuais de perdas mal-elaboradas, relações de dependência patológicas e estresse.

     

    Fonte: http://pepsic.bvsalud.org/scielo.php?script=sci_arttext&pid=S1516-36872005000100002


ID
1797775
Banca
CESPE / CEBRASPE
Órgão
TJ-DFT
Ano
2015
Provas
Disciplina
Psicologia
Assuntos

Com relação à atuação do profissional de psicologia nas questões relacionadas ao direito de família, da criança e do adolescente, julgue o item seguinte.

As intervenções psicológicas em situação de divórcio e separação, que passam pelo crivo das partes que poderão vir a contestar, criticar e por à prova o trabalho psicológico, em decorrência do princípio do contraditório e da livre convicção do juiz, têm por objetivo resgatar o embasamento teórico e o risco patológico desse momento, principalmente para os filhos.

Alternativas
Comentários
  • Tem por objetivo o bem-estar de todos, principalmente, o dos filhos, se houver, pois a regra principal é preservar o melhor interesse dos filhos.Jorge Trindade (Psicologia jurídica para operadores do direito, pg 189).

  •    Na esteira do espírito renovador da Constituição Federal de 1988 voltada para promoção e proteção dos direitos de terceira geração, assim reconhecidos legalmente como direitos difusos e coletivos, através de leis próprias e específicas para atender as diferenças ressaltadas pelo princípio da isonomia, no ano de 1990 foi promulgado a lei 8.069, que estabelece o Estatuto da criança e do Adolescente (ECA) 

     

      O ECA, que é um verdadeiro microssistema, cuida da proteção aos interesses de crianças e adolescentes, nos seus aspectos administrativos, penais e civis, esses últimos são o objetivo desse pequeno ensaio. 

     

      O ECA, como seu próprio nome diz, é um estatuto, que em linguagem técnica jurídica, refere-se a qualquer lei que disciplina direitos e deveres de uma específica categoria de determinadas pessoas. No caso do ECA, disciplina direitos e deveres de crianças e adolescentes. Para efeitos legais, crianças são todas as pessoas naturais que contam com até 12 anos de idade, incompletos. Adolescentes são pessoas com idade entre 12 e 18 anos. De acordo com o que dispõe o código civil crianças são absolutamente incapazes (de 12 a 16 anos) e relativamente capazes (de 16 a 18 anos). Em casos excepcionais aplicará o ECA para pessoas de idade 18 a 21 anos. 

     

    https://jus.com.br/artigos/25506/eca-aspectos-civis

  • Aplica-se o princípio do melhor interesse da criança e do adolescente

     

    ------------------- 

    Gabarito: Errado


ID
1832776
Banca
FGV
Órgão
TJ-RO
Ano
2015
Provas
Disciplina
Psicologia
Assuntos

Um juiz encaminha ao psicólogo uma criança cujos pais disputam a guarda para a realização de perícia. Com base nas resoluções CFP nº 017/2012, que dispõe sobre a atuação do psicólogo como perito nos diversos contextos, e CFP nº 008/2010, que dispõe sobre a atuação do psicólogo como perito e assistente técnico no Poder Judiciário, é correto afirmar que:

Alternativas
Comentários
  • A) Errada: o princípio do bem-estar é garantido a todos:Considerando que os psicólogos peritos e assistentes técnicos deverão fundamentar sua intervenção em referencial teórico, técnico e metodológico respaldados na ciência Psicológica,na ética e na legislação profisional, garantindo como princípio fundamental o bem-estar de todos os sujeitos envolvidos.Resolução CFP 08/2010

    B) Correta: Art. 4º – O periciado deve ser informado acerca dos motivos, das técnicas utilizadas, datas e local da avaliação pericial psicológica.Resolução 017/2012.

    C) Errada: Art. 4° Parágrafo único: Quando a pessoa atendida for criança, adolescente ou interdito, é necessária a apresentação de consentimento formal a ser dado por pelo menos um dos responsáveis legais.Resolução 017/2012.

    D) Errada:A devolutiva em situações de perícia não tem objetivo de fazer prognósticos, seu objetivo é reponder os quesitos.A perícia é realizada de maneira focal  Art.10 – A devolutiva do processo de avaliação deve direcionar-se para os resultados dos instrumentos e técnicas utilizados.Resolução 017/2012.

    E) Errada: os psicólogos têm a função básica de assessoria técnica para decisões judiciais, dando um aval científico às decisões judiciais, porém sem entrar no mérito legal

  • Letra A, além da 08/2010, também está na 017/2012:

    Art 3º–Conforme a especificidade de cada situação, o trabalho pericial poderácontemplar observações, entrevistas, visitas domiciliares e institucionais, aplicação detestes psicológicos, utilização de recursos lúdicos e outros instrumentos, métodos etécnicas reconhecidas pela ciência psicológica, garantindo como princípio fundamental o bem-estar de todos os sujeitos envolvidos.

  • RESOLUÇÃO CFP nº 017/2012

    Art.3º – Conforme a especificidade de cada situação, o trabalho pericial poderá contemplar observações, entrevistas, visitas domiciliares e institucionais, aplicação de testes psicológicos, utilização de recursos lúdicos e outros instrumentos, métodos e técnicas reconhecidas pela ciência psicológica, garantindo como princípio fundamental o bem-estar de todos os sujeitos envolvidos (A)

    Art. 4º – O periciado deve ser informado acerca dos motivos, das técnicas utilizadas, datas e local da avaliação pericial psicológica (B)

    § único: Quando a pessoa atendida for criança, adolescente ou interdito, é necessária a apresentação de consentimento formal a ser dado por pelo menos um dos responsáveis legais (C)

    Art.10 – A devolutiva do processo de avaliação deve direcionar-se para os resultados dos instrumentos e técnicas utilizados (D)

    RESOLUÇÃO CFP Nº 008/2010

    Art. 7º - Em seu relatório, o psicólogo perito apresentará indicativos pertinentes à sua investigação que possam diretamente subsidiar o Juiz na solicitação realizada, reconhecendo os limites legais de sua atuação profissional, sem adentrar nas decisões, que são exclusivas às atribuições dos magistrados (E)

    Gabarito: B

  • Gab B

    Art. 4º – O periciado deve ser informado acerca dos motivos, das técnicas utilizadas, datas e local da avaliação pericial psicológica.Resolução 017/2012.


ID
1832788
Banca
FGV
Órgão
TJ-RO
Ano
2015
Provas
Disciplina
Psicologia
Assuntos

A psicologia iniciou sua trajetória científica através do estudo experimental dos processos psicológicos, os “elementos da mente”. Contudo, ela não ficou restrita aos limites do laboratório, dando origem a testes e avaliações que pudessem ser aplicados em diversos locais e com várias pessoas ao mesmo tempo. Com a acumulação de saber sobre a percepção, motivação e emoção, memória, aquisição de hábitos, tempo de reação, etc., a psicologia realizou historicamente a primeira grande aproximação com o campo do direito com o principal objetivo de:

Alternativas
Comentários
  • O primeiro ramo da psicologia Forense a surgir foi a psicologia criminal, pois realiza estudos psicológicos de alguns dos tipos mais comuns de delinquentes e criminosos em geral, como, por exemplo, os psicopatas. 

     

    Fonte:https://pt.wikipedia.org/wiki/Psicologia_jur%C3%ADdica

  • A Psicologia Jurídica emergiu da Psicologia do Testemunho cuja prática, em âmbito internacional, ajudou a consolidar a Psicologia enquanto ciência, dada a necessidade de sua contribuição na comprovação da fidedignidade de testemunhos, principalmente com o surgimento e aplicação dos testes psicológicos, em meados do século XX, assim como o desenvolvimento de estudos sobre os funcionamentos dos interrogatórios, dos delitos, dos falsos testemunhos e falsas memórias etc., colaborando para a criação dos primeiros laboratórios de Psicologia.

    Disponível em: https://meuartigo.brasilescola.uol.com.br/psicologia/a-importancia-psicologia-juridica.htm